You are on page 1of 119

NCLEX PHARMA STRATEGY

Common strategies for solving pharmacology-related


questions for next-generation Nclex

1. Understand the drug's mechanism of action:


Knowing how a drug works can help you predict its effects and side effects. For example, if you
understand that beta-blockers block the effects of adrenaline, you can predict that they will
decrease heart rate and blood pressure.
2. Know the drug's pharmacokinetics:
Understanding the drug's absorption, distribution, metabolism, and excretion can help you predict
its onset, duration, and elimination. For example, if you know that a drug is highly protein- bound,
you can predict that it will have a long half-life and may interact with other protein-bound drugs.
3. Recognize adverse effects and contraindications:
Knowing the potential adverse effects and contraindications of a drug can help you identify when
it is not appropriate for a patient. For example, if a patient has a history of severe allergic reactions,
you should avoid administering a drug that has at high risk of causing anaphylaxis.
4. Apply critical thinking skills:
When answering pharmacology-related questions, it is important to use critical thinking skills to
evaluate the situation and make an informed decision. For example, if a patient is experiencing a
severe adverse reaction to a drug, you may need to stop administering the drug and provide
immediate medical attention.
5. Consider patient-specific factors:
When administering drugs, it is important to consider the patient's age, weight, medical history,
and other relevant factors. For example, if a patient has impaired renal function, you may need to
adjust the dose or frequency of a drug that is eliminated primarily by the kidneys.
6. Understand the drug's therapeutic indications:
Understanding the specific conditions or symptoms that a drug is used to treat can help you identify
when it is appropriate to administer it. For example, if a patient is experiencing symptoms of
depression, you may consider administering a selective serotonin reuptake inhibitor (SSRI).
7. Recognize drug interactions:
Knowing which drugs may interact with each other can help you identify potential drug-drug
interactions and avoid harmful combinations. For example, if a patient is taking a drug that inhibits
CYP3A4, you should avoid administering a drug that is metabolized by that enzyme.
8. Familiarize yourself with drug classes:
Knowing the general characteristics of drug classes can help you identify which drugs are
appropriate for specific conditions or situations. For example, if a patient is experiencing an acute
asthma attack, you may consider administering a short-acting beta agonist (SABA).
9. Be familiar with dosage calculations:
Knowing how to calculate drug dosages based on a patient's weight, age, and other factors can
help you ensure that the patient receives the correct dose. For example, if a patient weighs 70 kg
and the recommended dose is 10 mg/kg, you would calculate the dose as 700 mg.
9. Use reliable drug information sources:
When answering pharmacology-related questions, it is important to use reliable drug information
sources to ensure that you are providing accurate and up-to-date information. For example, you
may use drug reference books, online databases, or clinical practice guidelines to find information
about specific drugs or drug classes.
10. Consider patient adherence:
When selecting a medication, it is important to consider the patient's ability to adhere to the
prescribed regimen. For example, if a patient has difficulty remembering to take medications
regularly, you may consider prescribing a long- acting medication that only needs to be taken once
per day.
11. Recognize different routes of administration:
Knowing the different routes of administration for medications can help you identify which route
is appropriate for a particular patient or situation. For example, if a patient is experiencing severe
pain and needs immediate relief, you may consider administering the medication intravenously
rather than orally.
13. Be aware of medication errors: Knowing the most common types of medication errors can help
you identify potential errors and take steps to prevent them. For example, you may be aware that
look-alike/sound-alike medications can cause confusion, so you would take extra care to double-
check the medication name and dosage before administering it.
14. Consider cultural and linguistic factors:
When administering medications to patients from different cultural backgrounds or who speak
different languages, it is important to consider cultural and linguistic factors that may affect their
understanding and acceptance of the medication. For example, you may need to use a medical
interpreter to explain the medication and its side effects to a patient who speaks a different
language.
15. Understand medication safety:
Knowing how to ensure medication safety can help you identify potential risks and take steps to
prevent adverse events. For example, you may be aware that administering a medication via an
incorrect route or at an incorrect dose can lead to serious harm, so you would take extra care to
double- check the medication order and calculate the correct dosage before administering it.
16. Recognize the importance of medication reconciliation:
Knowing how to perform medication reconciliation can help you identify discrepancies and
ensure that patients receive the correct medications. For example, you may compare a patient's
current medication list to their medication orders to ensure that all medications are accounted for
and correctly prescribed.
17. Consider pregnancy and lactation:
When administering medications to pregnant or breastfeeding patients, it is important to consider
the potential risks and benefits to the patient and the fetus or infant For example, you may avoid
prescribing medications that are known to be teratogenic or have a high risk of causing harm to
the developing fetus.
18. Be aware of medication allergies:
Knowing how to identify medication allergies and take steps to prevent allergic reactions can help
you ensure patient safety. For example, you may ask patients about any known drug allergies
before administering medications and monitor them closely for signs of an allergic reaction.
19. Understand the difference between brand and generic medications:
Knowing the difference between brand and generic medications can help you identify potential
cost savings and ensure that patients receive the correct medication. For example, you may
prescribe a generic medication that has the same active ingredient as a brand-name medication to
reduce the cost for the patient.
20. Consider special populations:
When administering medications to special populations, such as pediatric or geriatric patients, it
is important to consider their unique physiological and pharmacological characteristics. For
example, you may adjust medication doses based on a patient's weight or age to ensure that they
receive an appropriate dose.
21. Be familiar with medication storage and handling;
Knowing how to store and handle medications can help you prevent degradation, contamination,
or other issues that may affect medication efficacy or safety For example, you may store
medications at the appropriate temperature, avoid exposing them to light or moisture, and use
aseptic techniques when preparing injectable medications.
22. Understand pharmacokinetics and pharmacodynamics:
Knowing the principles of pharmacokinetics and pharmacodynamics can help you understand
how medications are absorbed, distributed, metabolized, and eliminated in the body, as well as
their mechanism of action and therapeutic effects. For example, you may understand that a
medication that has a long half-life may require less frequent dosing than a medication with a short
half-life.
23. Be aware of medication side effects:
Knowing the potential side effects of medications can help you identify adverse reactions and take
appropriate measures to manage them. For example, you may monitor patients for common side
effects such as nausea, vomiting, dizziness, or rash, and report any serious or life-threatening
reactions immediately.
24. Understand medication withdrawal and dependence:
Knowing how medications can cause dependence or withdrawal symptoms can help you identify
patients who may need tapering or monitoring when discontinuing a medication. For example, you
may be aware that benzodiazepines can cause physical dependence and withdrawal symptoms, so
you would avoid abrupt discontinuation and consider tapering the medication over several weeks.
25. Consider medication administration devices:
Knowing how to use medication administration devices, such as inhalers, nebulizers, insulin pens,
or infusion pumps, can help you ensure that patients receive the correct dose and achieve optimal
therapeutic outcomes. For example, you may instruct patients on the correct use of an inhaler,
including how to prime the device, how to coordinate inhalation and activation, and how to clean
the device regularly.
26. Understand medication interactions:
Knowing how medications can interact with each other can help you identify potential drug
interactions and take appropriate measures to prevent adverse events. For example, you may be
aware that combining two medications that have a similar mechanism of action or are metabolized
by the same enzyme system can increase the risk of toxicity or adverse effects.
27. Consider the impact of comorbidities:
When prescribing medications to patients with comorbidities, it is important to consider the
potential impact on their other medical conditions and medication regimens. For example, you
may avoid prescribing medications that can exacerbate or interact with a patient's existing medical
conditions, such as hypertension or diabetes.
28. Be aware of medication dosing in renal or hepatic impairment:
Knowing how medications are metabolized and eliminated in patients with renal or hepatic
impairment can help you adjust medication dosing to prevent toxicity or adverse events. For
example, you may reduce the dose of a medication that is primarily eliminated by the kidneys in a
patient with renal impairment.
29. Consider medication cost and insurance coverage:
When selecting a medication, it is important to consider the cost and insurance coverage for the
patient. For example, you may prescribe a medication that is covered by the patient's insurance
plan or consider alternative medications that are less expensive or have a generic equivalent.
30. Understand medication tolerance and resistance:
Knowing how medications can lead to tolerance or resistance can help you identify patients who
may need alternative medications or dosage adjustments. For example, you may be aware that
long-term use of opioids can lead to tolerance, so you would monitor patients closely for signs of
decreased effectiveness or increasing doses needed.
31. Be aware of medication errors:
Knowing how to prevent and report medication errors can help you ensure patient safety and
quality of care. For example, you may double-check medication orders, verify patient identities
and medication allergies, and report any errors or near-misses immediately.
32. Understand the role of medication in disease management:
Knowing how medications are used to manage various diseases and conditions can help you select
appropriate medications and optimize therapeutic outcomes. For example, you may be aware that
beta-blockers are used to manage hypertension and prevent complications such as stroke or heart
attack.
33. Consider medication adherence and patient education:
When prescribing medications, it is important to consider patient adherence and provide
appropriate education on medication administration, side effects, and potential interactions. For
example, you may instruct patients on the importance of taking medications as prescribed, how to
store and handle medications, and how to report any side effects or concerns.
34. Be aware of medication use in pregnancy and breastfeeding:
Knowing how medications can affect pregnancy and breastfeeding can help you select appropriate
medications and ensure the safety of the mother and infant. For example, you may avoid
prescribing medications that are known to be harmful during pregnancy or breastfeeding, or
consider alternative medications that are safer.
35. Understand the ethical and legal implications of medication use:
Knowing the ethical and legal implications of medication use can help you make informed
decisions and advocate for patient rights and autonomy. For example, you may be aware of the
risks and benefits of prescribing medications off-label, or how to obtain informed consent for
medication administration.
Practice Questions
CASE STUDY:
Mrs. Jones is a 65-year-old female who has recently been diagnosed with hypertension. Her
medical history includes type 2 diabetes and hyperlipidemia. She has been prescribed a
combination drug therapy for her hypertension, which includes a calcium channel blocker and an
angiotensin- converting enzyme (ACE) inhibitor. Mrs. Jones has been experiencing mild dizziness
and headache since starting the medication therapy. She reports that she has not missed any doses
and has been taking the medication as prescribed.
1. What is the mechanism of action of the calcium channel blocker and the ACE inhibitor?
a) The calcium channel blocker blocks the effects of adrenaline, while the ACE inhibitor increases
blood pressure.
b) The calcium channel blocker blocks the effects of adrenaline, while the ACE inhibitor blocks
the renin- angiotensin-aldosterone system, which decreases blood pressure.
c) The calcium channel blocker blocks the renin-angiotensin- aldosterone system, which decreases
blood pressure, while the ACE inhibitor blocks the effects of adrenaline.
d) The calcium channel blocker blocks the renin- angiotensin-aldosterone system, which decreases
blood pressure, while the ACE inhibitor blocks angiotensin II, which also decreases blood
pressure.
Answer: d) The calcium channel blocker blocks the renin- angiotensin-aldosterone system, which
decreases blood pressure, while the ACE inhibitor blocks angiotensin II, which also decreases
blood pressure.

Explanation: Calcium channel blockers block calcium entry into cardiac muscle cells and smooth
muscle cells of the arterioles, leading to vasodilation and decreased blood pressure. ACE inhibitors
block the conversion of angiotensin I to angiotensin II, a potent vasoconstrictor, leading to
vasodilation and decreased blood pressure.

2. Which patient-specific factors should be considered when administering medication therapy for
hypertension?
a) Patient's weight and medical history
b) Patient's age and medical history
c) Patient's weight, medical history, and other relevant factors
d) Patient's age, medical history, and other relevant factors
Answer: c) Patient's weight, medical history, and other relevant factors
Explanation: When administering medications, it is important to consider patient-specific factors
such as age, weight, medical history, and other relevant factors. For example, in Mrs. Jones' case,
her medical history of diabetes and hyperlipidemia may impact her medication therapy for
hypertension.

Extended Drag and Drop Question:


Match the drug mechanism of action to the appropriate medication class:
 Calcium channel blockers
 Angiotensin-converting enzyme (ACE) inhibitors
 Beta-blockers
A Block the effects of adrenaline
B. Block the renin-angiotensin-aldosterone system, which decreases blood pressures
C. Block calcium entry into cardiac muscle cells and smooth muscle cells of the arterioles, leading
to vasodilation and decreased blood pressure
Answer:
 Calcium channel blockers - C. Block calcium entry into cardiac muscle cells and smooth
muscle cells of the arterioles, leading to vasodilation and decreased blood pressure
 Angiotensin-converting enzyme (ACE) inhibitors - B. Block the renin-angiotensin-
aldosterone system, which decreases blood pressure
 Beta-blockers - A. Block the effects of adrenaline

Case: A 68-year-old male patient presents to the emergency department with complaints of chest
pain. After performing an electrocardiogram (ECG), the physician diagnoses the patient with acute
myocardial infarction (AMI). The patient is started on aspirin and heparin and is scheduled to
undergo percutaneous coronary intervention (PCI) within the next few hours. The nurse
responsible for the patient's care should:
A) Administer an additional dose of aspirin
B) Administer an additional dose of heparin
C) Review the patient's medical history for potential contraindications to the medications
D) Administer a medication to relieve the patient's pain
Answer: C) Review the patient's medical history for potential contraindications to the medications
Explanation:

In this case, the nurse should review the patient's medical history to identify any potential
contraindications to the medications that have been prescribed, including aspirin and heparin.
Patients with a history of gastrointestinal bleeding or allergy to aspirin should not receive aspirin,
and patients with a history of heparin- induced thrombocytopenia should not receive heparin. The
nurse should also assess the patient for any potential adverse effects of the medications and monitor
the patient's response to the treatment.
A) Administering an additional dose of aspirin is incorrect because the patient has already been
started on aspirin, and administering an additional dose without reviewing the patient's medical
history could be harmful if there are any contraindications to aspirin.
B) Administering an additional dose of heparin is incorrect because the patient has already been
started on heparin, and administering an additional dose without reviewing the patient's medical
history could be harmful if there are any contraindications to heparin.
D) Administer a medication to relieve the patient's pain is incorrect because the patient is already
receiving medications for the treatment of AMI, and pain relief should not be the nurse's priority
in this case. The nurse should focus on ensuring that the patient is receiving appropriate
medications and monitoring the patient for potential adverse effects of the treatment.

Case: A 45-year-old patient presents to the emergency department with symptoms of chest pain
and shortness of breath. The patient's medical history includes hypertension, hyperlipidemia, and
type 2 diabetes. The healthcare provider orders several medications for the patient, including a
beta- blocker, a statin, and an ACE inhibitor.
What is the rationale for administering a beta-blocker to a patient with hypertension and chest
pain?
A) Beta-blockers increase heart rate and blood pressure
B) Beta-blockers decrease heart rate and blood pressure
C) Beta-blockers increase insulin sensitivity
D) Beta-blockers reduce blood glucose levels
Answer: B) Beta-blockers decrease heart rate and blood pressure.
Explanation: Beta-blockers work by blocking the effects of adrenaline, which reduces heart rate
and blood pressure. They are commonly prescribed for patients with hypertension, angina, and
heart failure. In this case, the patient has hypertension and is experiencing chest pain, which may
be a sign of angina or a heart attack. Administering a beta-blocker can help reduce the patient's
heart rate and blood pressure, which can improve blood flow to the heart and reduce the risk of
further cardiac events.
CASE: A 45-year-old male patient with a history of hypertension presents to the emergency
department with complaints of severe chest pain that radiates to his left arm. Upon evaluation, the
patient is diagnosed with acute myocardial infarction (heart attack). The physician orders several
medications, including nitroglycerin, aspirin, and heparin. As the nurse assigned to this patient,
which of the following is the most important factor to consider when administering these
medications?
A) Understanding the drugs' therapeutic indications
B) Knowing the drugs' pharmacokinetics
C) Recognizing potential adverse effects and contraindications
D) Considering the patient's medical history and other relevant factors
Answer: C) Recognizing potential adverse effects and contraindications
Explanation: When administering medications to a patient with acute myocardial infarction, it is
crucial to recognize potential adverse effects and contraindications of the drugs being
administered. Nitroglycerin is a potent vasodilator that can cause a significant drop in blood
pressure and can exacerbate bypotension in patients with underlying cardiovascular disease.
Aspirin is an antiplatelet agent that can increase the risk of bleeding, especially in patients with a
history of bleeding disorders or those taking anticoagulants such as heparin. Heparin is an
anticoagulant that can also increase the risk of bleeding and should be used with caution in patients
with a history of bleeding disorders, recent surgeries, or other contraindications. Therefore, it is
important for the nurse to recognize potential adverse effects and contraindications of these
medications to avoid further complications in the patient's condition.
When administering medications to a patient with acute myocardial infarction, which of the
following factors is the most important for the nurse to consider?
A) Understanding the drugs' therapeutic indications
B) Knowing the drugs' pharmacokinetics
C) Recognizing potential adverse effects and contraindications
D) Considering the patient's medical history and other relevant factors
Answer: C) Recognizing potential adverse effects and contraindications
Explanation: When administering medications to a patient with acute myocardial infarction, it is
crucial to recognize potential adverse effects and contraindications of the drugs being
administered. Nitroglycerin is a potent vasodilator that can cause a significant drop in blood
pressure and can exacerbate hypotension in patients with underlying cardiovascular disease.
Aspirin is an antiplatelet agent that can increase the risk of bleeding, especially in patients with a
history of bleeding disorders or those taking anticoagulants such as heparin. Heparin is an
anticoagulant that can also increase the risk of bleeding and should be used with caution in patients
with a history of bleeding disorders, recent surgeries, or other contraindications. Therefore, it is
important for the nurse to recognize potential adverse effects and contraindications of these
medications to avoid further complications in the patient's condition. Option A, B, and D are
important factors to consider when administering medications to a patient; however, recognizing
potential adverse effects and contraindications is the most important factor to consider in this
particular case.

Medicine side effects


1. Nausea and vomiting:
Some medications can cause gastrointestinal disturbances such as nausea and vomiting.
particularly when first starting or adjusting the medication dose. Example: Chemotherapy drugs
used to treat cancer, such as cisplatin, can cause nausea and vomiting as a side effect.
2. Diarrhea or constipation:
Some medications can cause changes in bowel habits, such as diarrhea or constipation. Example:
Opioid pain medications, such as oxycodone, can cause constipation as a common side effect.
3. Dizziness or lightheadedness:
Some medications can cause dizziness or lightheadedness, which can increase the risk of falls and
injuries. Example: Blood pressure medications, such as alpha- blockers, can cause a drop in blood
pressure and subsequent dizziness.
4. Sedation or drowsiness:
Some medications can cause sedation or drowsiness, which can affect a patient's ability to drive or
operate machinery. Example: Benzodiazepines, such as diazepam, are commonly used to treat
anxiety and can cause sedation as a side effect.
5. Headache:
Some medications can cause headaches as a side effect.
Example: Nitroglycerin, used to treat angina, can cause headaches as a common side effect.
6. Allergic reactions:
Some medications can cause allergic reactions, ranging from mild skin rashes to severe
anaphylaxis. Example: Penicillin antibiotics can cause allergic reactions in some patients,
including hives and difficulty breathing.
7. Dry mouth:
Some medications can cause dry mouth as n side effect. Example: Antidepressant medications,
such as tricyclic antidepressants, can cause dry mouth as a common side effect.
8. Insomnia:
Some medications can cause difficulty sleeping or insomnia as a side effect. Example: Stimulant
medications used to treat attention deficit hyperactivity disorder (ADHD), such as
methylphenidate, can cause difficulty sleeping as a side effect.
9. Muscle cramps or tremors:
Some medications can cause muscle cramps or tremors as a side effect. Example: Certain
medications used to treat Parkinson's disease, such as levodopa, can cause tremors as a common
side effect.
10. Rash:
Some medications can cause skin rashes or itching as a side effect.
Example: Antibiotics, such as sulfonamides, can cause a rash as a common side effect.

Practice Questions
CASE: Samantha is a 50-year-old woman who has recently been diagnosed with hypertension.
Her doctor prescribed her a medication called lisinopril, which is an ACE inhibitor used to lower
blood pressure. However, after taking the medication for a few days, Samantha complains of some
uneasiness in her health. Which side effects could be related to Samantha's medication?
A) Nausea and vomiting
B) Diarrhea or constipation
C) Dizziness or lightheadedness
D) Sedation or drowsiness
E) Headache
F) Allergic reactions
G) Dry mouth
H) Insomnia
I) Muscle cramps or tremors
J) Rash
Answer: C, G, and H
Explanation:
C) Dizziness or lightheadedness is a potential side effect of blood pressure medications, such as
ACE inhibitors like lisinopril. This can be due to a drop in blood pressure, which may increase the
risk of falls and injuries.
G) Dry mouth is also a common side effect of some blood pressure medications, including ACE
inhibitors. Samantha's complaint of dry mouth is consistent with this potential side effect.
H) Insomnia or difficulty sleeping can also be a side effect of some blood pressure medications,
such as ACE inhibitors. This may be due to the medication's effect on the body's systems that
regulate sleep.
A, B, E, F, I, and J are not related to Samantha's medication or her symptoms.

Case Study: A 60-year-old male patient with a history of chronic pain due to degenerative disc
disease was prescribed a drug for pain management. He has been taking the medication as directed
for the past two weeks but reports having difficulty with bowel movements. He describes his stools
as hard and infrequent, and he has only had one bowel movement in the past five days. He denies
any abdominal pain, bloating, or distension. Upon further questioning, he reports having a diet
high in protein and low in fiber.
MCQ Question: Which medication can cause constipation as a common side effect?
A) Chemotherapy drugs such as cisplatin
B) Blood pressure medications such as alpha-blockers
C) Benzodiazepines such as diazepam
D) Opioid pain medications such as oxycodone
Answer: D) Opioid pain medications such as oxycodone can cause constipation as a common side
effect.
Explanation: As mentioned in the article, some medications can cause changes in bowel habits,
such as diarrhea or constipation. Opioid pain medications, such as oxycodone, can cause
constipation as a common side effect. In the given case study, the patient has been prescribed
oxycodone for pain management and reports difficulty with bowel movements. This is consistent
with the known side effect of constipation associated with the medication. The other options in the
question, such as chemotherapy drugs, blood pressure medications, and benzodiazepines, are
associated with other side effects mentioned in the article, but not specifically constipation.
CASE: John is a 70-year-old male with a history of hypertension. He was recently started on a
new blood pressure medication, amlodipine. After a week of taking the medication, he reported
constipation. He had difficulty passing stools and felt bloated. He tried increasing his fiber and
fluid intake but did not see any improvement.
Which of the following statements is true regarding John's condition?
A) John's constipation is not related to his medication.
B) Amlodipine is known to cause constipation as a side effect.
C) John needs to stop taking his medication immediately.
D) John should increase his amlodipine dosage to relieve his constipation.
Answer: B) Amlodipine is known to cause constipation as a side effect.
Explanation: Option B is correct because some medications can cause changes in bowel habits,
such as constipation, as a side effect, and amlodipine is one of them. John's symptoms are
consistent with medication-induced constipation, which can occur due to the medication's effect
on the digestive system. Options A, C, and D are incorrect because constipation is a well- known
side effect of amlodipine, and John should not stop taking his medication without consulting his
healthcare provider or increase his dosage without medical advice.

Medication administration route


Medication administration route refers to the way in which medications are delivered into the body.
The various routes of medication administration include oral, topical, transdermal, inhalation,
intravenous (IV), intramuscular (IM), subcutaneous (SC), and rectal. Here are some examples of
medications administered through different routes:
1. Oral medication administration:
Oral medications are taken by mouth and are absorbed through the gastrointestinal tract.
Examples of oral medications include:
 Acetaminophen (Tylenol)
 Aspirin
 Metformin (Glucophage)
 Lisinopril (Zestril)

2. Topical medication administration:


Topical medications are applied directly to the skin or mucous membranes. Examples of topical
medications include:
 Lidocaine cream
 Hydrocortisone cream
 Clotrimazole cream (Lotrimin)

3. Transdermal medication administration:


Transdermal medications are delivered through the skin via a patch. Examples of transdermal
medications include:
 Nicotine patch
 Fentanyl patch (Duragesic)
 Scopolamine patch (Transderm Scop)

4. Inhalation medication administration:


Inhalation medications are delivered directly to the lungs through inhalation. Examples of
inhalation medications include:
 Albuterol inhaler (ProAir, Proventil)
 Ipratropium inhaler (Atrovent).
 Fluticasone inhaler (Flovent)

5. Intravenous (IV) medication administration:


IV medications are injected directly into a vein. Examples of IV medications include:
 Furosemide (Lasix)
 Dopamine
 Vancomycin (Vancocin)

6. Intramuscular (IM) medication administration:


IM medications are injected into a muscle. Examples of IM medications include:
 Penicillin G benzathine (Bicillin L-A)
 Tetanus toxoid (Td)
 Progesterone in oil (PIO)

7. Subcutaneous (SC) medication administration:


SC medications are injected under the skin. Examples of SC medications include:
 Insulin
 Enoxaparin (Lovenox).
 Epinephrine (EpiPen)

8. Rectal medication administration:
Rectal medications are inserted into the rectum. Examples of rectal medications include:
 Bisacodyl suppository (Dulcolax)
 Acetaminophen suppository
 Hydrocortisone suppository
It is important for nurses to have a thorough understanding of the different medication
administration routes and the appropriate medications for each route in order to ensure safe
and effective medication administration.

Practice Questions
--------------- is administered through the oral medication administration route?
a. Nicotine patch
b. Hydrocortisone cream
c. Acetaminophen (Tylenol) (Answer)
d. Albuterol inhaler
Explanation: Oral medications are taken by mouth and are absorbed through the gastrointestinal
tract. Examples of oral medications include Acetaminophen (Tylenol), Aspirin, Metformin
(Glucophage), and Lisinopril (Zestril).

--------------------------------administration route involves the delivery of medication through the


skin via a patch?
a. Intravenous (IV) medication administration
b. Topical medication administration
c. Intramuscular (IM) medication administration
d. Transdermal medication administration (Answer)
Explanation: Transdermal medications are delivered through the skin via a patch. Examples of
transdermal medications include Nicotine patch, Fentanyl patch (Duragesic), and Scopolamine
patch (Transderm Scop).
CASE: Mrs. Johnson is a 65-year-old patient who has been diagnosed with hypertension and has
been taking Lisinopril (Zestril) for the past six months. She has been complaining of dry cough
and dizziness for the past two weeks. Her healthcare provider decided to switch her medication to
a different route of administration.
Question 1: Which medication administration route is most likely to be chosen for Mrs. Johnson?
 Oral
 Topical
 Intravenous
 Subcutaneous
Answer: b) Topical
Explanation: Mrs. Johnson has been experiencing adverse effects of the medication through the
oral route, which is being absorbed through the gastrointestinal tract. Therefore, the healthcare
provider may switch her medication route to a topical route, which involves applying the
medication directly to the skin. Examples of topical medications include Lidocaine cream,
Hydrocortisone cream, and Clotrimazole cream.
Question 2: Which medication is administered through the rectal route?
a) Albuterol inhaler
b) Vancomycin (Vancocin)
c) Bisacodyl suppository (Dulcolax)
d) Fentanyl patch (Duragesic)
Answer: c) Bisacodyl suppository (Dulcolax)
Explanation: Rectal medication administration involves inserting medications into the rectum.
Examples of rectal medications include Bisacodyl suppository, Acetaminophen suppository, and
Hydrocortisone suppository.
Question 3: Which medication is administered through the intravenous route?
a) Albuterol inhaler
b) Furosemide (Lasix)
c) Hydrocortisone cream
d) Insulin
Answer: b) Furosemide (Lasix)
Explanation: Intravenous medication administration involves injecting medications directly into a
vein. Examples of intravenous medications include Furosemide (Lasix), Dopamine, and
Vancomycin (Vancocin).
Question 4: Which medication is administered through the subcutaneous route?
a) Enoxaparin (Lovenox)
b) Lisinopril (Zestril)
c) Acetaminophen (Tylenol)
d) Ipratropium inhaler (Atrovent)
Answer: a) Enoxaparin (Lovenox)
Explanation: Subcutaneous medication administration involves injecting medications under the
skin. Examples of subcutaneous medications include Insulin, Enoxaparin (Lovenox), and
Epinephrine (EpiPen).
Question 5: Which medication is administered through the transdermal route?
a) Nicotine patch
b) Penicillin G benzathine (Bicillin L-A)
c) Metformin (Glucophage)
d) Lisinopril (Zestril)
Answer: a) Nicotine patch
Explanation: Transdermal medication administration involves delivering medications through the
skin via a patch. Examples of transdermal medications include Nicotine patch, Fentanyl patch
(Duragesic), and Scopolamine patch (Transderm Scop).

Dosage Calculation
Dosage calculation is an essential skill for nurses, and it's important to be able to perform these
calculations accurately to ensure patient safety. Here are some of the main formulae for dosage
calculation that you may encounter on the NCLEX-RN exam:
Important Terms:
 Route: How the medicine is entering the body (IV. Oral, Topically)
 Medication: It's the actual compound that is completing the effect
 Dose: The amount of medicine/compound that is being administered.
 Desired dose: dosage that the health care provider prescribed
 Available dose: the dosage strength as stated on the medication label

Metric Table
 1,000 Micrograms (mcg)=1 milligram(mg)
 1,000 milligram(mg)=1 gram(gm)
 1,000 (gm)=1 kilogram(kg)
 1 kilogram=2.2 pound(lbs)
 30 milliliters(ml) =1 ounce(oz)
 1 milliliter (ml)= 1 cubic centimeter(ce)
 1,000 milliliters(ml)=1 liter(L)
 1 teaspoon(isp)=5 milliliters(ml)
 3 teaspoon(tsp)=1 tablespoon (Tbsp)
 1 tablespoon(Tbsp)=15 milliliters(ml)
 2 tablespoon(Tbsp)=1 ounce(oz)

1. Oral medicine -: Solid(tablets or capsule)


No. of tablets= (Desired/Available) x tablet(med level)
2. Oral/Parenteral Meds:- Liquid form
(Desired dose/stock dose) x Dilution (meds level)
3.IVF Rate:-ml/hr=(total vol (ml) x gtt factor (15))/no. of hours g
tts/min=vol in cc x gtt factor/ no. of hrsx 60 mins
3. Infusion Time= Total vol to infuse in ml/milliliters per hour being infused
 Working with pediatric dose
You can calculate a pediatric dose by dividing the child's body surface area (BSA) by the
average adult BSA (1.73 m2) and multiplying that number by the average adult dose of the
drug
PEDIATRIC DOSE-(BSA in m2/Average adult BSA (1.73 m2)) x Average adult dose

Pediatric dose =Child's body weight in kg Required number of mg of the drug per kg.

Other important formulae

1. Basic formula:
Dosage (Desired Dose/ Available Dose) x Route Conversion Factor
This formula is used to calculate the amount of medication that should be administered to the
patient based on the available dose and the desired dose.

2. IV rate calculation formula:


IV Rate (ml/hr) =Total Volume (ml) / Time (hr)
This formula is used to calculate the infusion rate for intravenous medications, such as fluids or
medications administered through a drip.

3. Pediatric dosage calculation formula:


Dosage= (Child's Weight in kg x Desired Dose) / Average Adult Dose
This formula is used to calculate the appropriate dose of medication for pediatric patients based
on their weight.
4. Reconstitution formula
Dose per ml = Total amount of medication needed / Total volume of diluent
This formula is used to calculate the concentration of a medication after it has been mixed with a
diluent.
5. Body surface area formula:

BSA=√ (Height (cm) x Weight (kg) - 3600)


This formula is used to calculate the body surface area of a patient, which is important when
determining medication dosages or radiation doses.

6. Flow rate calculation formula:


Flow Rate (drops per minute) = Volume (ml) x Drop Factor/ Time (min)
This formula is used to calculate the rate at which an IV solution should be administered in drops
per minute, based on the drop factor of the tubing.

7. Maintenance fluid calculation formula:


Maintenance Fluid (ml/hr) = Weight (kg) x 4
This formula is used to calculate the hourly rate of fluids that should be administered to a patient
to maintain hydration.
8. Titration formula:
Titration = (Desired Dose / Concentration) x Volume
This formula is used to calculate the amount of medication needed to achieve a desired
concentration, based on the volume of the medication being administered.

9. Dilution formula:
Dilution Factor = Volume of Stock Solution / Volume of Diluent
This formula is used to calculate the dilution factor needed to achieve a desired concentration of a
medication
10. Adjusted body weight formula:
Adjusted Body Weight (ABW)= Ideal Body Weight (IBW)+ 0.4 (Actual Weight - IBW)
This formula is used to calculate the appropriate medication dosage for overweight or obese
patients, as their actual weight may not accurately reflect their body composition.
11. Alligation formula:
Alligation (Strength of Solution A x Quantity of Solution A) + (Strength of Solution B x Quantity
of Solution B) Total Quantity of Solution
This formula is used to calculate the concentration of a medication solution when two or more
solutions with different strengths are mixed together.
12. Drip rate calculation formula:
Drip Rate (drops per minute) =Total Volume (ml) x Drop Factor/ Total Time (min)
This formula is used to calculate the drip rate for medications or fluids that are administered over
a specific period of time.
Remember to practice these formulae regularly and to understand the units of measurement used
in medication administration. Being able to accurately calculate medication dosages is an
important part of providing safe and effective patient care as a nurse.

Drug Interaction
When two or more OTC or prescription drug interact or react with each other making the
medicament either less effective or dangerous to health, it is called drug-drug interaction or simply
drug interaction. You may have heard that doctors often give special advice to people with high
blood pressure that they should avoid cold medications containing decongestants as this may cause
an increase in the patient's blood pressure.
How does NClex tests your knowledge of drug interaction?
They may give you some hypothetical questions about a patient with High Blood Pressure and a
patient approaching you for the advice regarding intake of medication during the cold.
One of the options could be to avoid using decongestants as far as possible.
Having said that, remember, a drug interaction is not always bad.
Some drug combinations can complement or increase the effect of others. Example: penicillin and
probenecid combination drug therapy.
We will go through some important examples of drug interactions in this chapter-
Example 1:
Probenecid blocks the excretion of penicillin so, penicillin stays in the blood for a longer period.
This is helpful as it can be helpful to decrease the dose or improve the efficacy of the dose of
penicillin given.
NClex doesn't test your deep knowledge of any specific drug but most important interactions and
drugs in general.
Example 2:
Do you know why Aspirin and codeine are given in combination?
They can give greater relief in pain management.
Taking about the interaction of aspirin with codeine, Let's talk a little bit about the adverse effects
too. It's safe to take aspirin as a painkiller with paracetamol or codeine. But we should be careful
with the administration of aspirin with ibuprofen or naproxen without talking to a doctor.
Example 3:
Aspirin, ibuprofen, and naproxen belong to the same group of medicines called non-steroidal anti-
inflammatory drugs (NSAIDS). The dose of aspirin may have to be reduced in such cases.
Question:
Which of the following is vital to be informed the patient if he is taking phenytoin?
Your options are:
i. Do not walk in the sun after taking the medicine
ii. Do not take milk at the same time.
iii. Do not swim until the dose is complete and the doctor advises you.
If you are unsure, phenytoin is used to control seizures (convulsions), including tonic-clonic (grand
mal) and psychomotor (temporal lobe) seizures, in the treatment of epilepsy. As a registered nurse,
you may be familiar with phenytoin administration basically during brain surgery as it can reduce
seizures.
The correct answer to this question is 'do not take milk at the same time. Nutritional supplements
and milk interfere with the absorption of phenytoin.
Example 4: I want to give you some other advice for phenytoin absorption.
 Phenytoin is compatible only with saline solutions; dextrose forms an insoluble
precipitate with phenytoin,
 Carbamazepine decreases blood levels of phenytoin and hormonal contraceptives.
Do not concomitantly administer phenytoin with carbamazepine.
 Phenytoin interacts with hormonal contraceptives one of the main reasons behind
the failure of hormonal contraceptives with carbamazepine is the drug interaction.
QUICK TIP
As soon as you see the drug phenytoin or Antacids or the term 'milk' in your examination,
remember there could be questions or chances of interaction.
Let us see some other cases where milk should not be administered to the patient concomitantly:
Example 5:
Tetracyclines can interact with calcium and magnesium thus causing an inadequate absorption of
tetracyclines. Milk contains calcium so we should be cautious in the administration of tetracyclines
with milk.
Examples of tetracyclines are lymecycline, methacycline, minocycline, rolitetracycline, and
doxycycline.
Example 6:
Do not give iron supplements such as ferrous gluconate with milk. - it interferes with drug
absorption.
Example 7:
Do not give milk when the patient is administered with charcoal as it affects its adsorption.
Activated charcoal has pores that can trap chemicals. It is typically taken by mouth as a treatment
for some swallowed poisons.
I have seen questions revolving around milk, charcoal, or antacid interaction with drugs in many
competitive nursing exams.
Interaction between drug with antacid-
Example 8: Give special care with digoxin administration and antacid as antacid decreases its
absorption.
Example 9:

Tetracycline interacts with magnesium and calcium and many antacid compounds contain
magnesium and calcium so it is very important to be extra cautious while giving tetracycline with
milk or antacids.
Example 10: With iron supplements such as Ferrous gluconate, we may not administer a patient
with antacid as this will affect its absorption.
Example 11: We should inform the patients to take extra precautions for the patients taking
metronidazole. Why? When mixed with alcohol, metronidazole causes a disulfi ram-like effect
involving nausea, vomiting, and other unpleasant symptoms.
Example 12: Another common interaction is with spironolactone which is a potassium-sparing
diuretic and potassium-rich food such as banana or spinach or salts. The client should avoid salt
substitutes because of the high potassium content while taking spironolactone.
Example 13: Not all interaction cases are harmful. They can be used to prevent certain adverse
reactions. Drug interactions are sometimes used to prevent or antagonize certain adverse reactions.
The diuretics hydrochlorothiazide and spironolactone are often given together because
hydrochlorothiazide is potassium- depleting and spironolactone is potassium-sparing.

Other Examples of Drug Interactions


1. Warfarin and NSAIDs: NSAIDS can increase the risk of bleeding when taken with warfarin,
a blood-thinning medication. Example: A patient taking warfarin for atrial fibrillation should avoid
taking aspirin or ibuprofen for pain relief.
2. Statins and macrolide antibiotics: Macrolide antibiotics can increase the risk of muscle
damage when taken with statins, which are used to lower cholesterol. Example: A patient taking
atorvastatin for high cholesterol should avoid taking erythromycin for an infection.
3. Digoxin and diuretics: Diuretics can cause electrolyte imbalances that can affect the
effectiveness of digoxin, which is used to treat heart failure. Example: A patient taking digoxin for
heart failure should be monitored for hypokalemia if also taking furosemide, a diuretic.
4. Benzodiazepines and opioids: Opioids can increase the risk of respiratory depression when
taken with benzodiazepines, which are used to treat anxiety and insomnia. Example: A patient
taking alprazolam for anxiety should avoid taking oxycodone for pain relief.
5. ACE inhibitors and potassium-sparing diuretics: Potassium-sparing diuretics can increase
potassium levels in the blood, which can be dangerous when taken with ACE inhibitors, which are
used to treat hypertension. Example: A patient taking lisinopril for hypertension should avoid
taking spironolactone, a potassium-sparing diuretic.
6. Lithium and diuretics: Diuretics can increase the risk of lithium toxicity by decreasing sodium
levels in the blood. Example: A patient taking lithium for bipolar disorder should avoid taking
hydrochlorothiazide, a diuretic.
7. Oral contraceptives and antibiotics: Antibiotics can decrease the effectiveness of oral
contraceptives, increasing the risk of pregnancy Example: A patient taking oral contraceptives
should use a backup method of contraception if also taking amoxicillin for an infection.
8. Methotrexate and nonsteroidal anti-inflammatory drugs (NSAIDs): NSAIDS can increase
the risk of methotrexate toxicity when taken together, as both medications are cleared by the
kidneys. Example: A patient taking methotrexate for rheumatoid arthritis should avoid taking
naproxen for pain relief.
9. Insulin and beta-blockers: Beta-blockers can mask the symptoms of hypoglycemia, making
it difficult for patients with diabetes to recognize low blood sugar levels. Example: A patient taking
metoprolol for hypertension should be monitored closely for signs of hypoglycemia if also taking
insulin for diabetes.
10. Warfarin and herbal supplements: Certain herbal supplements can interact with warfarin,
either increasing or decreasing its effectiveness. Example: A patient taking warfarin should avoid
taking ginkgo biloba, which can increase the risk of bleeding, and should consult with their
healthcare provider before taking any other herbal supplements.
11. Calcium channel blockers and grapefruit juice: Grapefruit juice can increase the levels of
some calcium channel blockers in the blood, leading to a greater risk of side effects. Example: A
patient taking verapamil for hypertension should avoid drinking grapefruit juice.
12. MAOIS and tyramine-containing foods: Foods containing tyramine can cause dangerous
spikes in blood pressure when taken with MAOIS, which are used to treat depression. Example: A
patient taking phenelzine for depression should avoid eating aged cheese, cured meats, and
fermented foods.
13. Anticoagulants and antiplatelet agents: Anticoagulants and antiplatelet agents can increase
the risk of bleeding when taken together. Example: A patient taking aspirin for cardiovascular
disease should be closely monitored if also taking warfarin.
14. Rifampin and oral contraceptives: Rifampin can decrease the effectiveness of oral
contraceptives by increasing their metabolism. Example: A patient taking rifampin for tuberculosis
should use a backup method of contraception if also taking oral contraceptives.
15. Theophylline and ciprofloxacin: Ciprofloxacin can increase the levels of theophylline in the
blood, leading to a greater risk of side effects. Example: A patient taking theophylline for asthma
should avoid taking ciprofloxacin for an infection.
16. Antacids and tetracyclines: Antacids can decrease the absorption of tetracyclines, making
them less effective. Example: A patient taking doxycycline for an infection should avoid taking
antacids.
17. ACE inhibitors and NSAIDs: NSAIDs can decrease the effectiveness of ACE inhibitors,
leading to poorly controlled hypertension.
Example: A patient taking enalapril for hypertension should avoid taking ibuprofen for pain relief.
18. Beta-blockers and verapamil: Verapamil can increase the levels of some beta-blockers in the
blood, leading to a greater risk of side effects. Example: A patient taking propranolol for
hypertension should avoid taking verapamil for angina.
19. Levodopa and pyridoxine: Pyridoxine can decrease the effectiveness of levodopa, which is
used to treat Parkinson's disease. Example: A patient taking levodopa for Parkinson's disease
should avoid taking high-dose pyridoxine supplements.
20. Warfarin and cranberry juice: Cranberry juice can interact with warfarin, either increasing
or decreasing its effectiveness Example: A patient taking warfarin should avoid drinking large
amounts of cranberry juice and should consult with their healthcare provider before consuming
any other cranberry products.
21. Calcium channel blockers and grapefruit juice: Grapefruit juice can increase the levels of
some calcium channel blockers in the blood, leading to a greater risk of side effects. Example: A
patient taking verapamil for hypertension should avoid drinking grapefruit juice.
22. MAOIS and tyramine-containing foods: Foods containing tyramine can cause dangerous
spikes in blood pressure when taken with MAOIS, which are used to treat depression. Example: A
patient taking phenelzine for depression should avoid eating aged cheese, cured meats, and
fermented foods.
23. Anticoagulants and antiplatelet agents: Anticoagulants and antiplatelet agents can increase
the risk of bleeding when taken together. Example: A patient taking aspirin for cardiovascular
disease should be closely monitored if also taking warfarin.
24. Rifampin and oral contraceptives: Rifampin can decrease the effectiveness of oral
contraceptives by increasing their metabolism. Example: A patient taking rifampin for tuberculosis
should use a backup method of contraception if also taking oral contraceptives.
25. Theophylline and ciprofloxacin: Ciprofloxacin can increase the levels of theophylline in the
blood, leading to a greater risk of side effects. Example: A patient taking theophylline for asthma
should avoid taking ciprofloxacin for an infection.
26. Antacids and tetracyclines: Antacids can decrease the absorption of tetracyclines, making
them less effective. Example: A patient taking doxycycline for an infection should avoid taking
antacids.
27. ACE inhibitors and NSAIDs: NSAIDs can decrease the effectiveness of ACE inhibitors,
leading to poorly controlled hypertension. Example: A patient taking enalapril for hypertension
should avoid taking ibuprofen for pain relief.
28. Beta-blockers and verapamil: Verapamil can increase the levels of some beta-blockers in the
blood, leading to a greater risk of side effects. Example: A patient taking propranolol for
hypertension should avoid taking verapamil for angina.

29. Levodopa and pyridoxine: Pyridoxine can decrease the effectiveness of levodopa, which is
used to treat Parkinson's disease. Example: A patient taking levodopa for Parkinson's disease
should avoid taking high-dose pyridoxine supplements.

Practice Questions
CASE: A 60-year-old patient has been diagnosed with hypertension and is prescribed lisinopril,
an ACE inhibitor, for its management. However, the patient also experiences edema and is
prescribed spironolactone, a potassium-sparing diuretic. After a week of treatment, the patient
presents with confusion, muscle weakness, and fatigue. The doctor suspects hyperkalemia and
reviews the patient's medication history.
Question: What is the interaction between lisinopril and spironolactone?
A) Potassium-sparing diuretics can decrease the effectiveness of ACE inhibitors.
B) ACE inhibitors can decrease the effectiveness of potassium- sparing diuretics.
C) Potassium-sparing diuretics can increase potassium levels in the blood, leading to hyperkalemia
when taken with ACE inhibitors.
D) ACE inhibitors can cause hypokalemia when taken with potassium-sparing diuretics.
Answer: C
Explanation: Spironolactone, a potassium-sparing diuretic, can increase potassium levels in the
blood. This can be dangerous when taken with ACE inhibitors, like lisinopril, which can also
increase potassium levels. Therefore, taking these medications together can result in hyperkalemia,
which can cause symptoms like confusion, muscle weakness, and fatigue.
CASE: Ms. Johnson is a 70-year-old patient who suffers from hypertension,
hypercholesterolemia, and osteoarthritis. She is currently taking lisinopril for hypertension,
atorvastatin for hypercholesterolemia, and naproxen for osteoarthritis. She has recently developed
a urinary tract infection and her healthcare provider prescribed her ciprofloxacin for treatment.
However, the healthcare provider noticed that Ms. Johnson's medication list includes medications
that can interact with ciprofloxacin. The healthcare provider decided to adjust Ms. Johnson's
medication regimen to prevent potential drug interactions and adverse effects.
What is the potential drug interaction between lisinopril and ciprofloxacin?
A. Lisinopril can increase the risk of bleeding when taken with ciprofloxacin.
B. Lisinopril can decrease the effectiveness of ciprofloxacin.
C. Lisinopril can increase the risk of muscle damage when taken with ciprofloxacin.
D. Lisinopril can cause electrolyte imbalances when taken with ciprofloxacin.
Answer: B. Lisinopril can decrease the effectiveness of ciprofloxacin. Lisinopril is an ACE
inhibitor used to treat hypertension, and it can decrease the effectiveness of ciprofloxacin, an
antibiotic used to treat infections.
What is the potential drug interaction between atorvastatin and ciprofloxacin?
A. Atorvastatin can increase the risk of bleeding when taken with ciprofloxacin.
B. Atorvastatin can decrease the effectiveness of ciprofloxacin.
C. Atorvastatin can increase the risk of muscle damage when taken with ciprofloxacin.
D. Atorvastatin can cause electrolyte imbalances when taken with ciprofloxacin.
Answer: C. Atorvastatin can increase the risk of muscle damage when taken with ciprofloxacin.
Ciprofloxacin can increase the levels of atorvastatin in the blood, leading to a greater risk of muscle
damage.
Which of the following medication combinations can lead to poorly controlled hypertension?
a) Antacids and tetracyclines
b) ACE inhibitors and NSAIDs
c) Beta-blockers and verapamil
d) Levodopa and pyridoxine
Answer: b) ACE inhibitors and NSAIDs. NSAIDs can decrease the effectiveness of ACE
inhibitors, leading to poorly controlled hypertension.
Which of the following medication combinations can increase the risk of bleeding when taken
together?
a) Theophylline and ciprofloxacin
b) Anticoagulants and antiplatelet agents
c) Beta-blockers and verapamil
d) Rifampin and oral contraceptives
Answer: b) Anticoagulants and antiplatelet agents. Anticoagulants and antiplatelet agents can
increase the risk of bleeding when taken together.
Which of the following medication combinations can decrease the effectiveness of levodopa?
a) Antacids and tetracyclines
b) Levodopa and pyridoxine
c) Warfarin and cranberry juice
d) MAOIS and tyramine-containing foods
Answer: b) Levodopa and pyridoxine. Pyridoxine can decrease the effectiveness of levodopa,
which is used to treat Parkinson's disease.
Which of the following medication combinations can interact with each other to either increase or
decrease effectiveness?
a) Theophylline and ciprofloxacin
b) Anticoagulants and antiplatelet agents
c) Warfarin and cranberry juice
d) ACE inhibitors and NSAIDs
Answer: c) Warfarin and cranberry juice. Cranberry juice can interact with warfarin, either
increasing or decreasing its effectiveness.

Medicine Contraindications
1. Allergy: A contraindication to a medication is an allergy to the medication or any of its
components. For example, someone who is allergic to penicillin should not take any penicillin-
based antibiotics such as amoxicillin or augmentin.
2. Pregnancy: Some medications are contraindicated during pregnancy as they may harm the
developing fetus. For example, thalidomide is contraindicated in pregnancy as it causes birth
defects.
3. Breastfeeding: Some medications can pass into breast milk and may harm the nursing infant.
For example, aspirin should not be taken while breastfeeding as it can cause bleeding disorders in
infants.
4. Age: Some medications may not be safe for use in certain age groups. For example, aspirin
should not be given to children under the age of 16 due to the risk of Reye's syndrome.
5. Pre-existing medical conditions: Certain medical conditions may make certain medications
unsafe or contraindicated. For example, beta-blockers are contraindicated in patients with asthma
due to their potential to exacerbate symptoms.
6. Drug interactions: Certain medications may interact with other drugs a person is taking, making
them unsafe or contraindicated. For example, some antibiotics can interact with birth control pills,
making them less effective.

7. Liver or kidney disease: Some medications may not be safe for use in patients with liver or
kidney disease. For example, acetaminophen should be used with caution in patients with liver
disease as it can cause liver damage.
8. Psychiatric conditions: Certain medications may not be safe for use in patients with psychiatric
conditions. For example, stimulants used to treat ADHD may exacerbate symptoms in patients
with anxiety disorders.
9. Blood disorders: Some medications may be contraindicated in patients with blood disorders.
For example, warfarin should not be used in patients with a history of bleeding disorders or blood
clots.
11. Immunodeficiency: Some medications may be contraindicated in patients with
immunodeficiency, as they may increase the risk of infections. For example, live vaccines are
contraindicated in patients with immunodeficiency, as they contain live viruses that can cause
infections.
12. Cardiovascular disease: Some medications may be contraindicated in patients with
cardiovascular disease, as they may worsen the condition. For example, nonsteroidal anti-
inflammatory drugs (NSAIDs) should be used with caution in patients with heart failure or high
blood pressure.
13. Gastrointestinal conditions: Some medications may be contraindicated in patients with
gastrointestinal conditions. For example, aspirin should not be used in patients with a history of
stomach ulcers or gastrointestinal bleeding.
14. Seizure disorder: Some medications may be contraindicated in patients with seizure disorders,
as they may lower the seizure threshold. For example, bupropion should not be used in patients
with a history of seizures.
14. Glaucoma: Some medications may be contraindicated in patients with glaucoma, as they may
increase intraocular pressure. For example, antihistamines should be used with caution in patients
with glaucoma.
15. Thyroid disorder: Some medications may be contraindicated in patients with thyroid disorders.
For example, lithium should not be used in patients with hyperthyroidism, as it may worsen the
condition.
16. Hepatitis: Some medications may be contraindicated in patients with hepatitis, as they may
cause liver damage. For example, statins should be used with caution in patients with hepatitis, as
they may increase liver enzymes.
17. Diabetes: Some medications may be contraindicated in patients with diabetes. For example,
corticosteroids should be used with caution in patients with diabetes, as they may increase blood
sugar levels.
18. Renal impairment: Some medications may be contraindicated in patients with renal
impairment. For example, aminoglycoside antibiotics should be used with caution in patients with
renal impairment, as they may cause kidney damage.
19. Autoimmune disease: Some medications may be contraindicated in patients with autoimmune
disease, as they may worsen the condition.For example, interferon should not be used in patients
with autoimmune diseases such as lupus.
20. Mental health conditions: Some medications may be contraindicated in patients with mental
health conditions. For example, benzodiazepines should not be used in patients with a history of
substance abuse or addiction, as they may be addictive.

Practice Questions
What is the potential risk of using beta-blockers in patients with asthma?
A) Exacerbation of asthma symptoms
B) Liver damage
C) Bleeding disorders
D) Increased risk of infections
Answer: A) Exacerbation of asthma symptoms.
Explanation: Beta-blockers are contraindicated in patients with asthma as they can exacerbate
symptoms.
A 45-year-old female with a history of hypertension, type 2 diabetes, and bipolar disorder presents
to her primary care physician with symptoms of a urinary tract infection. The physician prescribes
ciprofloxacin for the infection. The patient asks if there are any concerns about the medication
interacting with her medical history.
Which of the following conditions may be a contraindication for the use of ciprofloxacin?
A) Immunodeficiency
B) Gastrointestinal conditions
C) Diabetes
D) Renal impairment
Answer: D) Renal impairment
Explanation: Renal impairment can be a contraindication for ciprofloxacin as it can cause kidney
damage. Patients with renal impairment should use caution when taking aminoglycoside
antibiotics, including ciprofloxacin. Immunodeficiency, gastrointestinal conditions, and diabetes
are not specific contraindications for ciprofloxacin.

Case Study: A 55-year-old male patient visits the clinic complaining of joint pain. He has a history
of hypertension, type 2 diabetes, and renal impairment. His current medications include
metformin, lisinopril, and amlodipine. The physician prescribes ibuprofen for his joint pain. What
is a potential contraindication for ibuprofen in this patient?
A) Immunodeficiency
B) Cardiovascular disease
C) Renal impairment
D) Autoimmune disease
Answer: C) Renal impairment
Explanation: Some medications may be contraindicated in patients with renal impairment.
Nonsteroidal anti-inflammatory drugs (NSAIDs) such as ibuprofen should be used with caution in
patients with renal impairment, as they may cause kidney damage. In this case, the patient has a
history of renal impairment and is already taking medications for hypertension and diabetes, which
puts him at higher risk for kidney damage. Therefore, ibuprofen should not be prescribed without
careful consideration of its potential risks and benefits in this patient. Options A, B, and D are not
relevant in this case.

Pneumonics and Memory


1. ABCDE Rule: Used to prioritize emergency treatment for drugs that cause toxicity. A for
Airway, B for Breathing. C for Circulation, D for Disability, and E for Exposure.
2. ADME: Absorption, Distribution, Metabolism, and Excretion. This acronym reminds you of the
pharmacokinetics of drugs.
3. "COLDSPA": Character, Onset, Location, Duration, Severity, Pattern, and Associated Factors.
This mnemonic helps you remember the important aspects of assessing pain.
4. "SLUDGE": Salivation, Lacrimation, Urination, Defecation, Gastrointestinal Disturbances, and
Emesis. This acronym reminds you of the symptoms of cholinergic toxicity.
5. "DUMBBELLS": Diarrhea, Urination, Miosis, Bradycardia, Bronchospasm, Emesis,
Lacrimation, and Sweating. This mnemonic helps you remember the symptoms of cholinergic
toxicity.
6. "PINCH": Pallor, Pain, Paresthesia, Paralysis, Poikilothermia, Pulselessness. This acronym
helps you remember the symptoms of arterial occlusion.
7. "BEEF": Bones, Epithelium, Eyes, Fat, and Fetus. This acronym reminds you of the tissues that
are affected by vitamin A toxicity.
8. "COPD": Chronic Obstructive Pulmonary Disease. This acronym reminds you of the respiratory
condition that is treated with bronchodilators.
9. "CHF": Congestive Heart Failure. This acronym reminds you of the condition that is treated
with diuretics.
10. "ARDS: Acute Respiratory Distress Syndrome. This acronym reminds you of the condition
that is treated with oxygen therapy.
11. "CAT: Cardiovascular, Anti-Inflammatory, and Thrombolytic. This acronym reminds you of
the types of drugs that are used to treat cardiovascular disease.
12. "MAG": Magnesium, Anti-Arrhythmic, and Glaucoma. This acronym reminds you of the types
of drugs that affect the heart and eyes.
13. "HALT": High Alert Medication, Assess Patient, Label Properly, and Teach Patients. This
acronym reminds you of the steps you need to take when administering high-risk medications.
14. "CAP": Community-Acquired Pneumonia. This acronym reminds you of the type of
pneumonia that is treated with antibiotics.
15. "ATI": Anticholinergic Toxicity, Tachycardia, and Ileus. This acronym reminds you of the
symptoms of anticholinergic toxicity.
16. "BRASH": Bleeding, Rash, Anemia, Sore Throat, and Headache. This acronym reminds you
of the adverse effects of heparin.
17. "NEVER": Not Ever Mix or Dilute with Anything. Verify before administering, Ensure proper
storage and preparation, Read the label and follow instructions. This acronym reminds you of the
steps you need to take when administering medication.
18. "PIN": Pain, Infection, Nutrition. This acronym reminds you of the important aspects of patient
care.
19. "PRIME": Prevent, Reduce, Increase, Monitor, and Evaluate. This acronym reminds you of the
steps you need to take when managing medication therapy.
20. "RIGHT": Right Patient, Right Drug, Right Dose, Right Route, Right Time, Right
Documentation. This acronym reminds you of the six rights of medication administration.
21. "DISH": Drugs, Infection, Stress, and Hydration. This acronym reminds you of the factors that
can affect the metabolism of medication.
22 "CATS". Calcium, Antihistamines, Thyroid medication, and Steroids. This acronym reminds
you of the medications that can cause osteoporosis.
23. "FAST": Facial droop, Arm weakness, Speech difficulties, and Time. This acronym reminds
you of the signs of a stroke.
24. "CHEATED": Cold and clammy skin, Hypotension, ECG changes, Anxiety, Thready pulse,
Elevated temperature, and Diaphoresis. This acronym reminds you of the signs of shock.
25. "OHIO": Opioids, Hypnotics, Inhalants, and Other medications. This acronym reminds you
of the types of medications that can cause respiratory depression.
26. "PIGS": Penicillin, Insulin, Gentamicin, and Streptomycin. This acronym reminds you of the
medications that require monitoring of renal function.

27. "STEPS: Salicylates, Tinnitus, Elevated temperature, Psychosis, and Seizures. This acronym
reminds you of the symptoms of salicylate toxicity.
28. "SEA": Stupor, Excitement, and Analgesia. This acronym reminds you of the stages of
anesthesia.
29. "PAD" Peripheral Arterial Disease. This acronym reminds you of the condition that is treated
with vasodilators.
30, "CAT: Cardiovascular, Antihypertensive, and Thrombolytic. This acronym reminds you of the
types of drugs that are used to treat cardiovascular disease.
31. "PEA": Pulseless Electrical Activity. This acronym reminds you of the condition that is treated
with CPR and epinephrine.
32. "PQRST" Provocative or Palliative factors, Quality of pain, Region and Radiation of pain,
Severity of pain, and Timing of pain. This acronym helps you remember the important aspects of
assessing pain.
33. "MILK": Morphine, Insulin, Lasix, and K-CL. This acronym reminds you of the medications
that should be drawn up first when preparing multiple medications in one syringe.
34, "FACES": Fear, Anxiety, Communication, Environment, and Support System. This acronym
reminds you of the important aspects of patient care.
35. "CRASH": Chest pain, Respiratory distress, Altered mental status, Severe bleeding or shock,
and Head injury. This acronym reminds you of the medical emergencies that require immediate
intervention.

Patient education and counseling


Antibiotics: Patients prescribed antibiotics should be educated on the importance of taking the full
course of antibiotics as prescribed, even if they start feeling better. This is because stopping
antibiotics prematurely can lead to antibiotic resistance, where the bacteria becomes resistant to
the antibiotic For example, patients prescribed amoxicillin for a bacterial infection should be
instructed to take the full course of antibiotics, even if their symptoms improve after a few days.
Antidepressants: Patients prescribed antidepressants should be counseled on the possible side
effects of the medication and how to manage them.
For example, patients prescribed selective serotonin reuptake inhibitors (SSRIs) may experience
nausea, dizziness, or sexual dysfunction. Patients should be informed that these side effects usually
resolve after a few weeks of treatment and that they should not stop taking the medication without
first consulting their healthcare provider.
Beta blockers: Patients prescribed beta blockers should be educated on the importance of
monitoring their blood pressure and heart rate regularly.
For example, patients prescribed metoprolol for hypertension should be instructed to check their
blood pressure at home regularly and report any significant changes to their healthcare provider.
Insulin: Patients prescribed insulin should be counseled on proper administration techniques, such
as injection site rotation and proper dosage calculations. For example, patients with type I diabetes
who require insulin injections should be instructed to rotate injection sites to prevent
lipohypertrophy, which is a buildup of fat at the injection site.
Opioids: Patients prescribed opioids should be educated on the risks of addiction and overdose,
as well as the importance of proper storage and disposal of the medication.
For example, patients prescribed oxycodone for pain management should be instructed to keep the
medication in a secure location, away from children and pets, and to dispose of any unused
medication properly to prevent accidental ingestion or misuse.
Anticoagulants: Patients prescribed anticoagulants should be counseled on the signs and
symptoms of bleeding and the importance of monitoring for signs of bleeding.
For example, patients prescribed warfarin should be instructed to monitor for signs of bruising,
bleeding gums, or blood in their urine or stool, and to report any signs of bleeding immediately to
their healthcare provider.
Antihypertensives: Patients prescribed antihypertensives should be educated on the importance
of monitoring their blood pressure regularly and following lifestyle modifications such as a low-
sodium diet, regular exercise, and stress reduction.
For example, patients prescribed lisinopril for hypertension should be instructed to monitor their
blood pressure at home regularly and to make lifestyle modifications such as reducing their salt
intake and increasing physical activity.
Bronchodilators: Patients prescribed bronchodilators should be counseled on proper inhaler
technique and the importance of using a spacer.
For example, patients prescribed albuterol for asthma should be instructed on how to use their
inhaler properly and how to use a spacer to ensure that the medication is delivered effectively to
the lungs.
Antidiabetic agents: Patients prescribed antidiabetic agents should be educated on the importance
of monitoring their blood glucose levels regularly, following a healthy diet, and engaging in regular
physical activity.
For example, patients with type 2 diabetes prescribed metformin should be instructed on how to
monitor their blood glucose levels and to make lifestyle modifications such as following a low-
carbohydrate diet and engaging in regular physical activity.
NSAIDS: Patients prescribed NSAIDs should be counseled on the possible side effects of the
medication, such as stomach ulcers and kidney damage, and how to manage them.
For example, patients prescribed ibuprofen for pain management should be instructed to take the
medication with food to reduce the risk of stomach ulcers and to monitor for signs of kidney
damage, such as decreased urine output or swelling in the legs or feet.
Antihistamines: Patients prescribed antihistamines should be counseled on the possible side
effects of the medication, such as drowsiness, dry mouth, and constipation.
For example, patients prescribed diphenhydramine for allergies should be instructed to avoid
activities that require mental alertness, such as driving or operating machinery, until they know
how the medication affects them.
Statins: Patients prescribed statins should be educated on the importance of monitoring their
cholesterol levels regularly and making lifestyle modifications such as following a low-fat diet and
engaging in regular physical activity
For example, patients prescribed atorvastatin for high cholesterol should be instructed to monitor
their cholesterol levels regularly and to make lifestyle modifications such as reducing their intake
of saturated and trans fats and increasing their intake of fruits and vegetables.
Antipsychotics: Patients prescribed antipsychotics should be counseled on the possible side
effects of the medication, such weight gain, metabolic changes, and movement disorders.
For example, patients prescribed risperidone for schizophrenia should be instructed to monitor
their weight and to report any significant changes to their healthcare provider.
Diuretics: Patients prescribed diuretics should be educated on the importance of monitoring their
fluid and electrolyte balance and the possible side effects of the medication, such as dehydration
and low potassium levels.
For example, patients prescribed furosemide for heart failure should be instructed to monitor their
fluid intake and output and to report any significant changes to their healthcare provider.
Proton pump inhibitors: Patients prescribed proton pump inhibitors should be counseled on the
possible side effects of the medication, such as stomach upset and diarrhea, and how to manage
them.
For example, patients prescribed omeprazole for acid reflux should be instructed to take the
medication with food to reduce the risk of stomach upset and to monitor for signs of diarrhea or
other gastrointestinal side effects.
Opioids: Patients prescribed opioids should be counseled on the possible side effects of the
medication, such as drowsiness, constipation, and respiratory depression.
For example, patients prescribed oxycodone for pain management should be instructed to avoid
activities that require mental alertness, such as driving or operating machinery, until they know
how the medication affects them. They should also be instructed to monitor for signs of
constipation and to manage it with lifestyle modifications and medication if necessary.
Benzodiazepines: Patients prescribed benzodiazepines should be educated on the possible side
effects of the medication, such as drowsiness, confusion, and dependence.
For example, patients prescribed lorazepam for anxiety should be instructed to avoid activities that
require mental alertness until they know how the medication affects them. They should also be
instructed to take the medication as directed and not to exceed the prescribed dose.
Antidepressants: Patients prescribed antidepressants should be counseled on the possible side
effects of the medication, such as nausea, insomnia, and sexual dysfunction.
For example, patients prescribed sertraline for depression should be instructed to take the
medication as directed and to report any significant side effects to their healthcare provider.
Immunomodulators: Patients prescribed immunomodulators should be educated on the
importance of monitoring for signs of infection and the possible side effects of the medication,
such as headache, fatigue, and fever.
For example, patients prescribed adalimumab for autoimmune diseases should be instructed to
monitor for signs of infection, such as fever, cough, or sore throat, and to report any significant
changes to their healthcare provider.
Practice Questions
Mrs. Smith, a 62-year-old woman, was diagnosed with atrial fibrillation and prescribed warfarin.
She was counseled on the signs and symptoms of bleeding and the importance of monitoring for
signs of bleeding. However, after two weeks of taking the medication, she noticed bruising on her
arms and legs but didn't report it to her healthcare provider because she thought it was a normal
side effect. One day, she fell and suffered a minor head injury, which caused bleeding in her brain.
She was rushed to the hospital, where she was diagnosed with a hemorrhagic stroke.
Which of the following is true regarding patients prescribed anticoagulants?
A) They should be educated on proper insulin administration techniques.
B) They should be counseled on the possible side effects of the medication.
C) They should monitor for signs and symptoms of bleeding and report them to their healthcare
provider.
D) They should be educated on the risks of addiction and overdose.
Answer: C
Explanation: Patients prescribed anticoagulants should be counseled on the signs and symptoms
of bleeding and the importance of monitoring for signs of bleeding.
For example, patients prescribed warfarin should be instructed to monitor for signs of bruising,
bleeding gums, or blood in their urine or stool, and to report any signs of bleeding immediately to
their healthcare provider. Option A is incorrect because it pertains to insulin administration, not
anticoagulants. Option B is incorrect because it refers to counseling on possible side effects, which
is more relevant for antidepressants than anticoagulants. Option D is incorrect because it pertains
to the risks of opioids, not anticoagulants.

CASE: Samantha, a 55-year-old female with a history of heart failure, is prescribed furosemide
by her healthcare provider to manage her symptoms. As her virtual assistant, you are responsible
for educating her on the medication and its possible side effects.
Question: What should Samantha be instructed to monitor while taking furosemide for heart
failure?
A) Blood glucose levels
B) Fluid intake and output
C) Respiratory rate
D) Heart rate
Answer: B) Fluid intake and output
Explanation. Patients prescribed diuretics like furosemide should be instructed to monitor their
fluid intake and output and to report any significant changes to their healthcare provider. This is
because diuretics can cause dehydration and electrolyte imbalances, which can worsen heart failure
symptoms. Monitoring fluid intake and output can help patients and their healthcare providers
adjust the medication dosage as needed. Blood glucose levels, respiratory rate, and heart rate are
not directly related to the management of heart failure with furosemide.
QUE: Patients prescribed proton pump inhibitors should be counseled on:
A) The importance of monitoring their fluid and electrolyte balance
B) The possible side effects of the medication, such as drowsiness and constipation
C) The possible side effects of the medication, such as stomach upset and diarrhea
D) The importance of avoiding activities that require mental alertness
Answer: C) The possible side effects of the medication, such as stomach upset and diarrhea
Explanation: Patients prescribed proton pump inhibitors should be counseled on the possible
side effects of the medication, such as stomach upset and diarrhea, and how to manage them.
Proton pump inhibitors reduce the production of stomach acid and can sometimes cause
gastrointestinal side effects. Monitoring fluid and electrolyte balance is not typically necessary
for patients taking proton pump inhibitors. Drowsiness and constipation are side effects more
commonly associated with opioids and benzodiazepines. Avoiding activities that require
mental alertness is a precaution associated with opioids and benzodiazepines.

Antibiotics
Welcome to the new chapter of pharmacology theory session for nclex exam preparation. As we
have always dealt, we will not go in detail with every classification and mechanism of action for
all the drugs but will study the main or the most important points which are important for your
nclex preparation.

Penicillin
These are one of the earliest antibiotics discovered and are also very popular in use. So, how can
you identify these antibiotics?
Hint: Look for the ending 'cillin' at the end
Examples:
 amoxicillin,
 ampicillin,
 carbenicillin,
 nafcillin,
 oxacillin,
 dicloxacillin,
 piperacillin etc.
Side effects and nursing considerations:
Gl disturbance: The primary concern with penicillins that are orally administered is GI
disturbance so most penicillins taken orally are advised to take food and not on the empty stomach.
Allergic or hypersensitivity reactions (which present with rashes and anaphylaxis). Anaphylaxis
means severe and potentially life-threatening allergic reactions.
Change of resistance: Any antibiotics must be given in a full dose or full regimen so that their
effect is met and there is a decrease in drug resistance
Renal Excretion: concomitant use of other drugs may increase or decrease renal excretion. Also
if there is any renal disease, the drug excretion is reduced.
Pregnancy: It is advised not to take penicillin because it can affect the development of the infant
or can cause superinfections
Superinfections: any signs of superinfections should be monitored closely. Superinfections
simply are the infection following a previous infection especially when caused by microorganisms
that are resistant or have become resistant to the antibiotics used earlier.
Cephalosporins
To identify cephalosporin, look or start with 'cef' at the beginning of every drug name example:
 cefazolin,
 cefotaxime,
 cephalexin,
 cefdinir,
 cefaclor,
 cefadroxil etc.

The side effects and nursing consideration in the administration of this class of antibiotics is very
similar to penicillins. These drugs can have a cross-reaction with other cephalosporins and
penicillin drugs so care should be taken for this point too.

Important points
 Similar to penicillin, these cause Gl disturbance so should be taken with food.
 Care should be taken so that superinfections are reduced.
 There is a chance of hypersensitivity or allergic reactions.
 These drugs too have renal excretion and are nephrotoxic in patients who have predisposing
renal insufficiency.
 These drugs should be used with caution with pregnant and lactating mothers.
Aminoglycosides
As an identifying feature, look for the end moiety of this drug 'micin' examples are:
 Gentamicin
 Neomycin
 vancomycin
 tobramycin
 amikacin etc.
So, as we always do, we study the main adverse effects and nursing management for those
effects which are important for nclex exam point of view.

IMP: Aminoglycosides are Nephrotoxic and Ototoxic


 All the aminoglycosides are nephrotoxic and ototoxic. By ototoxic we mean that they can
cause hearing or hearing balance problems.
 As these are nephrotoxic we should access the kidney function through BUN, creatinine,
and urine output levels.
 They too can cause superinfection. Example of superinfection is Clostridium difficile
overgrowth.
 As these are not well absorbed from GI tract, they are usually given via the intravascular
or intramuscular route.
Red Man Syndrome: One important point with one of the drugs of this class, vancomycin is red
man syndrome. There is rash on the face, neck, and upper torso after intravenous administration
of vancomycin. It is an allergic reaction that is caused by the overstimulation of specific immune
cells in the body in response to vancomycin. If red man syndrome appears then the vancomycin
infusion should be discontinued immediately.
We give Diphenhydramine 60 to 9- minutes before administration of vancomycin to prevent this
condition.
They must also be given in a full dose or full regimen so that its effect is met and there is a decrease
in drug resistance.

Fluoroquinolone
How can we identify these classes of antibiotics?
look for the ending with 'xacin':
Examples:
 gatifloxacin,
 ciprofloxacin,
 iprofloxacin,
 gemifloxacin,
 levofloxacin,
 moxifloxacin,
 ofloxacin, etc.

Important points:
 Thrombocytopenia (bone marrow depression).
 There can be an immediate drop in the blood pressure, especially if they are given on an
IV line so keep this point in mind.
 Although there is Gl effect as with other antibiotics, remember, fluoroquinolone cannot be
taken with food.
 For quick memory, remember the pneumonic, FLU MA T-Fluoroquinolone, macrolides,
and Tetracyclines are the antibiotics that cannot be given with the food.
 Also, note there are photosensitivity reactions with this drug. So patient must be advised
not to walk under the sun.
 Other symptoms are quite common for every patient such as dizziness, headaches,
insomnia, depression, etc.

Sulfonamides
Like fluoroquinolone, the other class of drug that can cause thrombocytopenia or bone marrow
depression is sulfonamides,
These can be identified with the ending zine at the end of the drug
Examples are:
 Sulfasalazine
 Sulfadiazine
Common points as we have already discussed may cause bone marrow depression. This is also
called thrombocytopenia.
These classes of drugs are both nephrotoxin and hepatotoxic. So, we should access the liver
function tests and kidney function through BUN, creatinine, and urine output levels.
 Hepatotoxic. Monitor liver function tests (AST, ALT).
 Nephrotoxic. Monitor BUN, creatinine, and urine output levels
Photosensitive reactions: The patients must be informed to walk under the sun with care. Patients
must be advised to use sunscreen and protective clothing with sun exposure.
They can cause birth defects so shouldn't be used in pregnancy and lactation.
There can be cross-sensitivity and allergic reactions with drags of this class and with other
antibiotics.

Macrolides
The next group of antibiotics are macrolides and usually end with 'thromycin' or 'mycin' at the end.
Examples are:
 Erythromycin,
 Clarithromycin,
 Azithromycin and
 Thromycin
Note: Please do not get carried away with the thinking that all drugs ending with mycin are
macrolides. We also studied that drugs of the aminoglycoside group such as gentamycin,
vancomycin, or neomycin also end with the ending mycin. Watch out for 'thromycin' ending.
 Let's now see some of the main points for the drugs of this class
 Like with penicillins and cephalosporin and other antibiotics, they can cause Gl effects.
 They can cause superinfections. ALT). and may ultimately lead to cardiac death.
 They are hepatotoxic, Monitor liver function tests (AST)
 On ECG strip they can cause prolonged QT elongation LOTI
Note: Macrolyde can increase drug levels of certain anticoagulants such as warfarin,
carbamazepine, and theophylline. So, it is advised that patients already in these drugs should not
be given macrolides.

Tetracyclines
The drugs in this class come with the ending cycline
Examples are:
 omethacycline,
 minocycline,
 rolitetracycline,
 lymecycline a
 doxycycline.
Let's study some main points from drugs in this class.
 Drugs of this class stain the teeth
 Drugs of this class can also damage the bone.
 They are quite common for pill-induced esophagitis so patients must be advised to stay
upright after ingestion of this drug.
 These drugs can cause photosensitivity reactions. So, you should advice patients for
walking with sun- protective clothes and creams
 They can also cause hypersensitivity reactions. They also can form a complex with milk,
and avoid milk with tetracycline.
 As with all other antibiotics, they are GI intolerant but these drugs shouldn't be given with
food.
Strategies for solving antibiotic-related questions with relevant examples
1. Understand the mechanism of action of antibiotics: Knowing how an antibiotic works can help
you understand how it affects the body and how it can be used to treat different types of infections.
For example, penicillin works by disrupting the bacterial cell wall, leading to cell death. This
makes it effective against Gram-positive bacteria, which have a thick peptidoglycan layer in their
cell walls.
2. Memorize the names and classifications of antibiotics: There are many different classes of
antibiotics, each with its own unique properties and uses. It's important to be able to recognize
these drugs by name and classification. For example, tetracyclines are broad-spectrum antibiotics
that can be used to treat a wide range of bacterial infections, including acne, urinary tract
infections, and respiratory infections.
3. Understand the indications and contraindications for antibiotics: Antibiotics are powerful drugs
that can have serious side effects if used improperly. It's important to know when and how to use
antibiotics, as well as when they should be avoided. For example, fluoroquinolones should not be
used in children or pregnant women due to the risk of joint and tendon damage.
4. Know the common adverse effects of antibiotics: All medications have potential side effects,
and antibiotics are no exception. Being familiar with the most common adverse effects of
antibiotics can help you identify potential complications and monitor patients for signs of adverse
reactions.
For example, aminoglycosides can cause ototoxicity and nephrotoxicity, so patients taking these
drugs should be closely monitored for changes in hearing and kidney function.
Example: A patient is prescribed amoxicillin for a suspected bacterial infection. What should the
nurse monitor for?
Answer: The nurse should monitor the patient for signs of an allergic reaction, such as rash,
itching, or difficulty breathing. The nurse should also monitor the patient's gastrointestinal system
for signs of antibiotic-associated diarrhea, such as abdominal cramping and loose stools. If the
patient experiences any adverse effects, the nurse should report them to the healthcare provider
immediately.
5.Understand the importance of culture and sensitivity testing: Culture and sensitivity testing can
help identify the specific bacteria causing an infection and determine which antibiotic is most
effective against it. Nurses should understand how to collect specimens for testing and how to
interpret the results.
6. For example, if a patient has a suspected urinary tract infection, a urine culture and sensitivity
test can help determine if the infection is caused by a specific type of bacteria, such as E. coli, and
which antibiotics are most effective against it.Be familiar with dosing and administration
guidelines for antibiotics: Different antibiotics have different dosing and administration guidelines,
which can affect their effectiveness and safety. Nurses should be familiar with these guidelines and
ensure that patients receive the correct dose at the correct intervals.
For example, some antibiotics, such as azithromycin, are dosed differently for children than for
adults, so nurses should be aware of these differences when administering the drug.
7. Understand the concept of antimicrobial resistance: Antimicrobial resistance is a growing public
health concern that can lead to treatment failure and the spread of drug-resistant bacteria. Nurses
should be aware of the factors that contribute to antimicrobial resistance, such as overuse and
misuse of antibiotics, and take steps to prevent the development of resistance.
For example, nurses can educate patients about the importance of taking antibiotics only as
prescribed and completing the full course of treatment.
Example: A patient is prescribed ceftriaxone for a suspected respiratory infection. What nursing
assessments should be performed before and during treatment?
Answer: Before starting treatment with ceftriaxone, the nurse should assess the patient's medical
history and allergy history to ensure that the drug is safe for use. The nurse should also assess the
patient's vital signs, lung sounds, and oxygen saturation levels to establish a baseline and monitor
for signs of respiratory distress. During treatment, the nurse should monitor the patient for signs
of allergic reaction, such as rash, itching, or difficulty breathing, and report any adverse effects to
the healthcare provider immediately. The nurse should also ensure that the patient is receiving the
correct dose at the correct intervals and complete the full course of treatment to prevent the
development of antibiotic resistance.
8. Understand the different routes of administration for antibiotics: Antibiotics can be administered
via different routes, such as oral, intravenous, or intramuscular. Knowing the different routes of
administration can help nurses select the most appropriate route for the patient and ensure proper
administration.
For example, intravenous administration may be necessary for critically ill patients with severe
infections, while oral administration may be appropriate for patients with mild to moderate
infections who can tolerate oral medications.
9. Be aware of drug interactions with antibiotics: Some medications can interact with
antibiotics, leading to decreased effectiveness or increased risk of adverse effects, Nurses
should be aware of potential drug interactions and assess patients for concurrent medication
use.
For example, tetracyclines can interact with antacids and dairy products, which can decrease the
absorption of the drug and reduce its effectiveness
10. Understand the principles of infection control Infection control measures are important for
preventing the spread of antibiotic-resistan bacteria and protecting patients and healthcare
workers from infections. Nurses should understand the principles of infection control, such
as hand hygiene, standard precautions, and isolation precautions, and follow these
measures consistently For example, nurses should wash their hands before and after caring
for patients with infectious diseases, use personal protective equipment as appropriate, and
follow isolation protocols to prevent the spread of infections.
Example: A patient is prescribed vancomycin for a suspected infection. What nursing
assessments should be performed before and during treatment?
Answer: Before starting treatment with vancomycin, the nurse should assess the patient's renal
function, as vancomycin is primarily eliminated by the kidneys and can accumulate in patients
with impaired renal function. The nurse should also assess the patient's medical history and
allergy history to ensure that the drug is safe for use. During treatment, the nurse should
monitor the patient's kidney function, electrolyte levels, and vital signs, and report any adverse
effects to the healthcare provider immediately. The nurse should also ensure that the patient is
receiving the correct dose at the correct intervals and complete the full course of treatment to
prevent the development of antibiotic resistance. Finally, the nurse should follow infection
control protocols, such as hand hygiene and isolation precautions, to prevent the spread of
infection.

NSAIDS
The other class of drugs that we are going to learn about is NSAIDS. Unlike other drugs that have
a common ending, there are many drugs in this class so we will have to learn the main drug
properties for your nclex examination.
These are:
antipyretic- meaning they reduce the temperature and hence basically used in fever,
anti-inflammatory - they reduce inflammation and are hence used in various inflammatory
conditions like rheumatoid arthritis.
Let's see some common examples of NSAIDs.
 aspirin,
 ibuprofen,
 naproxen,
 ketorolac,
 indomethacin,
 diclofenac,
 aceclofenac,
 fenoprofen,
 ketoprofen, etc.

PARACETAMOL ACETAMINOPHEN
Remember, paracetamol is usually called acetaminophen in US so please keep in mind that
whenever you are asked about acetaminophen remember it is NSAIDS and paracetamol.
Note for confusing terms: Ketorolac is used for short-term management of moderately severe
acute pain that otherwise would require narcotics. Ketoprofen is used for the management of the
signs and symptoms of rheumatoid arthritis, osteoarthritis, and primary dysmenorrhea or menstrual
cramps.

Uses of NSAIDS:
 They AID in pain and inflammation reduction.
 It is anti-inflammatory - given to patients with rheumatoid arthritis.
Note: NSAIDs do not care about arthritis at all. They just case the pain of the patient and reduce
inflammation. Remember the term 'pain' whenever you hear of NSAIDs.
See these important points that could be beneficial for your nelex examination:
 These drugs shouldn't be given to patients with asthma because they can exacerbate the
condition.
 Whenever it is prescribed doctor prescribes you to take NSAIDS along with food so that it
can prevent you from Gastrointestinal upset.
 Gl bleeding is widely prevalent with these drugs. If a patient is already on anticoagulants
whose chance of bleeding is very high and at the same time you give NSAIDS to that
patient, the result could exacerbate the bleeding. If you are giving drugs like warfarin or
heparin make sure not to give any NSAIDs at the same time.

Interaction with lithium:


Lithium is used for people with bipolar disorders. NSAIDS are contraindicated in patients taking
lithium. This is because lithium is cleared through the renal route. NSAIDs decrease renal blood
flow. So, it can cause lithium toxicity. The characteristics of lithium toxicity are confusion, ataxia,
agitation, neuromuscular excitability, polyuria, and polydipsia. NSAIDs are nephrotoxic so people
with renal failure or disorder should avoid taking any NSAIDs.
Hepatotoxicity: NSAIDS are also hepatotoxic. There could be chances of severe hepatotoxicity
with NSAIDS basically with paracetamol. Remember, that you won't get any questions with
paracetamol because this is called acetaminophen in US. In the rest parts of the world basically,
acetaminophen is popular as paracetamol.
Sodium Retention: Another important point for your nelex preparation is sodium retention which
causes hypertension. Prolonged hypertension can result in congestive heart failure so make sure
you do not give NSAIDs to people with CHF and Hypertension.
Drug-Drug Interaction: With beta blockers, there is a decreased antihypertensive effect.
Remember the 'lol' drugs are beta-blockers, such as atenolol, propranolol, etc and these are given
for hypertension. Concomitant use of NSAIDs with these drugs decreases the hypertensive effect.
NSAIDS decrease the diuretic effect of loop diuretics.
Aspirin
Aspirin is also NSAIDs and is a derivative of salicylic acid. It has anti-inflammatory and
antipyretic effects. Aspirin toxicity Aspirin toxicity is called salicylism which presets symptoms
such as sweating, tinnitus, and headache It can also be used to prevent blood clot formation, and
hence prescribed to patients with some heart diseases and also for angina (or chest pain). There is
a chance of acute liver failure and encephalopathy seen in children who take aspirin making them
lethargic with vomiting and may lead to come. This is called Reye's syndrome.
OPIOIDS
Opioids are a family of drugs that are usually prescribed to Opioids a relieve pain, control coughs
and diarrhea, and treat addiction to other opioids. Note that, unlike NSAIDs, opioids don't reduce
fever and inflammation.
Whenever you think of Opioids, remember Opoid contains O like that in LOW
It lowers respiration, low brain activity 'sedation', and decreases the heart rate. That is- it has a
bradycardia effect.

Remember 'one' or 'ine' at the end of some common opioids like, oxycodone, hydrocodone,
codeine, and morphine.

Opioid Toxicity
In opioid toxicity, there is reverse respiratory depression, Tachycardia, Hypertension, Tremors, and
sweating. The main toxic effect is decreased respiratory rate and depth, which can progress to
apnea. Other complications (eg, pulmonary edema, which usually develops within minutes to a
few hours after opioid overdose) and death result primarily from hypoxia. Pupils are miotic in
opioid toxicity. So, whenever a patient is suffering from opioid toxicity, we have to administer
opioid antagonists such as naloxone and naltrexone. Naloxone can reverse the effects of other
opioids, such as heroin, morphine, and oxycodone. Naloxone thus reverses an opioid overdose.
Side effects and dealing with opioid toxicity
Side effects of opioids and mainly codeine which is also an opioid include constipation so we give
laxatives along with this drug.
Opioids cause respiratory depression and should be used widely with people with underlying
COPD or Chronic Obstructive Pulmonary Disorder Elderly people and also people who smoke too
much.
Strategies to solve questions of NSAIDs and Opioid family
To master pharmacology and medication administration questions for Next Generation NCLEX
RN, it is important to have a strong foundation in pharmacology concepts, drug classifications,
and medication administration. Here are some strategies that can help you prepare for questions
related to NSAIDS and opioids:
1. Understanding pharmacodynamics and pharmacokinetics: It is important to have a clear
understanding of how a drug works in the body (pharmacodynamics) and how it is
absorbed, distributed, metabolized, and excreted (pharmacokinetics). This will help you
predict how the drug will affect the patient and anticipate potential adverse effects or drug
interactions.
Example: A patient with chronic back pain is prescribed ibuprofen (NSAID) for pain relief. As an
RN, you should know that NSAIDs work by inhibiting the production of prostaglandins, which
are responsible for pain and inflammation. You should also be aware that ibuprofen is primarily
metabolized by the liver and excreted by the kidneys. Therefore, you should monitor the patient's
liver and kidney function and advise the patient to avoid taking ibuprofen with other drugs that
can cause liver or kidney damage.
2. Knowing drug classifications: Knowing the classification of a drug can help you
understand its mechanism of action, potential side effects, and interactions with other
drugs.
Example: A patient is prescribed morphine (opioid) for postoperative pain management. As an RN,
you should know that opioids work by binding to opioid receptors in the brain and spinal cord,
which can cause sedation, respiratory depression, and constipation. You should also be aware that
morphine is classified as a Schedule II drug, which means it has a high potential for abuse and
dependence. Therefore, you should closely monitor the patient for signs of respiratory depression
and opioid-related adverse effects, as well as for signs of drag seeking behavior or diversion.
3. Familiarizing yourself with medication administration: You should know the correct
dosage, route, and frequency of medication administration, as well as any special
considerations or precautions related to the medication.
Example: A patient is prescribed fentanyl (opioid) patches for chronic pain management.
As an RN, you should know that fentanyl is a potent opioid and should be administered
with caution, especially in elderly or debilitated patients or those with respiratory or hepatic
impairment. You should also be aware that fentanyl patches should be applied to intact skin
on a hairless area of the body and should not be cut or chewed. Therefore, you should
educate the patient on proper patch application and disposal, as well as monitor the patient
for signs of respiratory depression or overdose.

4. Understanding pharmacodynamics and pharmacokinetics: It is important to have a clear


understanding of how a drug works in the body (pharmacodynamics) and how it is
absorbed, distributed. metabolized, and excreted (pharmacokinetics). This will help you
predict how the drug will affect the patient and anticipate potential adverse effects or drug
interactions.
Example: A patient with chronic back pain is prescribed ibuprofen (NSAID) for pain
relief. As an RN, you should know that NSAIDs work by inhibiting the production of
prostaglandins, which are responsible for pain and inflammation. You should also be aware
that ibuprofen is primarily metabolized by the liver and excreted by the kidneys. Therefore,
you should monitor the patient's liver and kidney function and advise the patient to avoid
taking ibuprofen with other drugs that can cause liver or kidney damage.
5. Knowing drug classifications: Knowing the classification of a drug can help you
understand its mechanism of action, potential side effects, and interactions with other
drugs.
Example: A patient is prescribed morphine (opioid) for postoperative pain management.
As an RN, you should know that opioids work by binding to opioid receptors in the brain
and spinal cord, which can cause sedation, respiratory depression. and constipation. You
should also be aware that morphine is classified as a Schedule II drug, which means it has
a high potential for abuse and dependence. Therefore, you should closely monitor the
patient for signs of respiratory depression and opioid-related adverse effects, as well as for
signs of drug- seeking behavior or diversion.

6. Familiarizing yourself with medication administration: You should know the correct
dosage, route, and frequency of medication administration, as well as any special
considerations or precautions related to the medication.
Example: A patient is prescribed fentanyl (opioid) patches for chronic pain management.
As an RN, you should know that fentanyl is a potent opioid and should be administered
with caution, especially in elderly or debilitated patients or those with respiratory or hepatic
impairment. You should also be aware that fentanyl patches should be applied to intact skin
on a hairless area of the body and should not be cut or chewed Therefore, you should
educate the patient on proper patch application and disposal, as well as monitor the patient
for signs of respiratory depression or overdose.

7. Assessing patient understanding and adherence: Patient education is a critical component


of medication administration, and RNs should assess the patient's understanding and
adherence to medication regimens.
Example: A patient is prescribed tramadol (opioid) for chronic pain management. As an
RN, you should assess the patient's understanding of the medication's indications, side
effects, and potential risks, such as respiratory depression or addiction. You should also
assess the patient's adherence to the medication regimen and address any barriers or
concerns the patient may have, such as cost or fear of side effects.

8. Monitoring for adverse effects: NSAIDs and opioids can have serious adverse effects, and
RNs should monitor patients closely for signs of toxicity or overdose.
Example: A patient is prescribed acetaminophen and hydrocodone (opioid) for
postoperative pain management. As an RN, you should monitor the patient's liver function
and educate the patient on the maximum daily dose of acetaminophen to prevent liver
toxicity. You should also monitor the patient's respiratory rate and mental status for signs
of opioid overdose and administer naloxone if necessary.

9. Collaborating with interdisciplinary team members: RNs should collaborate with


physicians, pharmacists, and other healthcare team members to ensure safe and effective
medication management,
Example: A patient with chronic pain is prescribed oxycodone (opioid) by a pain
management physician. As an RN, you should collaborate with the physician and
pharmacist to ensure the patient's pain is adequately managed while minimizing the risk of
adverse effects or addiction. You should also communicate with other healthcare team
members, such as physical therapists or social workers, to address the patient's holistic
needs.

Antihypertensive drugs
In this chapter, we are studying drugs that are very common and end in lol' and 'pine' and these
are the antihypertensive drugs There are many classes of antihypertensive drugs. We are not going
to study each of them. Please also see the strategy section after this chapter to get in detail
knowledge.
Drugs ending in lol such as propranolol, and metoprolol are beta blockers.
Examples are: .
 propanalol, 
 metoprolol,  
 atenolol, 
 metoprolol  
 nadolol 
 pindolol  
 propranolol,  
 molol etc. 

These are used to treat dysrhythmias and decrease blood pressure by slowing down the heart rate.
Let's look at some important points for drugs in this class:
 Timolol is a beta blocker medication used either by mouth or as eye drops. As eye drops it
is used to treat increased pressure inside the eye such as in ocular hypertension and
glaucoma.
 Atenolol and metoprolol are cardioselective beta blockers and these are safer for people
with lung disorders.
Nonselective beta blockers have broncho-constrictive effects so they must be used cautiously
with patients who have co-existing asthma or chronic obstructive pulmonary disease (COPD)
because of the effects on Beta-2 receptors that could potentially cause bronchoconstriction.
So, what are the non-selective beta blockers?
propranolol, nadolol, pindolol, labetalol, penbutolol, sotalol, carvedilol

✔ Propranolol must never be used concomitantly with patients with COPD.

✓ Propranolol may cause bradycardia and nursing intervention for informing patients about
possible bradycardia effects is important for beta blockers.
ECG waves
Let's revise on ECG too. Because one way or another, they may test your knowledge on a beta
blocker and ECG interpretation. You should watch for QT width which is less than 0.4 seconds for
bradycardia. Other all waves are similar to normal sinus waves.
 P wave width 0.12 to 0.2 seconds.
 QRS width 0.04 to 0.12 seconds.
 QT width < 0.4 seconds.

 Beta-blockers should be slowly tapered as they have withdrawal effects like ischemia,
rebound hypertension, etc.
 Among all beta blockers, labetolol is safer in pregnancy. Methyldopa and labetolol are used
to treat blood pressure during pregnancy. Methyldopa is not a beta blocker but Methyldopa
is to treat high blood pressure in pregnancy.
 Methyldopa, has the lowest risk of harming the mother and developing fetus. Another
possible safe option for pregnant woman is diuretic.
Remember, methyldopa, diuretic and abetalol for hypertension in pregnancy (safest).
Calcium channel blocker
Note that calcium channel blockers ending drugs with the end- pine.
 Nifedipine
 Amlodipine
 Felodipine.
 Isradipine
 Nicardipine
 Nisoldipine
Just remember- 'pine' ending for calcium channel blockers.

✓ Calcium channel blockers hinder the influx of calcium into myocardial cells. We know, calcium
influx is needed in muscle fibers be it cardiac or other muscle fibers so contract.
✔So, stopping calcium influx via the calcium channel blockers can help dilate the blood vessels
or stop heart muscles in contracting. They may also increase the diameter of blood vessels, which
always results in decreased blood pressure. This is called vasodilation.
ECG waves:
Calcium channel blocker blocks calcium channels in the SA and AV nodes. IF SA node is blocked,
it decreases the heart rate and if AV node is blocked it decreases the speed of impulse conduction.
So, in conditions with very high heart rate like supraventricular tachycardia and atrial fibrillation,
we can use the calcium channel blockers.
Other points:
1. Remember, you should't take grapefruit with calcium channel blockers as it will increase the
level of calcium channel blockers in blood. This can cause hypotension
2. If you are already on antihypertensive drugs there are no other antihypertensive drugs that can
combine and interact to produce severe hypotension.
 It should also not be given with Diuretics (water pills)
 Interaction- calcium channel blockers can increase the digoxin level in the blood causing
digoxin toxicity. Similarly, it may show interactions with large amounts of vitamin D or
calcium supplements. Oral Care: The arteries dilate with calcium channel blockers. This is
true for some other muscles also, like gums. The gums swell. And this is called gingival
hyperplasia. So if you are on a calcium channel blocker, you may need to take good care
of oral health.
Strategies for solving anti-hypertensive drug-related questions

Case: Mrs. Jones is a 65-year-old female who has recently been diagnosed with hypertension. Her
medical history includes type 2 diabetes and hyperlipidemia. She has been prescribed a
combination drug therapy for her hypertension, which includes a calcium channel blocker and an
angiotensin-converting enzyme (ACE) inhibitor. Mrs. Jones has been experiencing mild dizziness
and headache since starting the medication therapy. She reports that she has not missed any doses
and has been taking the medication as prescribed.
1. What is the mechanism of action of the calcium channel blocker and the ACE inhibitor?
a) The calcium channel blocker blocks the effects of adrenaline, while the ACE inhibitor increases
blood pressure.
b) The calcium channel blocker blocks the effects of adrenaline, while the ACE inhibitor blocks
the renin- angiotensin-aldosterone system, which decreases blood pressure.
c) The calcium channel blocker blocks the renin-angiotensin- aldosterone system, which decreases
blood pressure, while the ACE inhibitor blocks the effects of adrenaline.
d) The calcium channel blocker blocks the renin- angiotensin-aldosterone system, which decreases
blood pressure, while the ACE inhibitor blocks angiotensin II. which also decreases blood
pressure.
Answer: d) The calcium channel blocker blocks the renin- angiotensin-aldosterone system, which
decreases blood pressure, while the ACE inhibitor blocks angiotensin II, which also decreases
blood pressure.
Explanation: Calcium channel blockers block calcium entry into cardiac muscle cells and smooth
muscle cells of the arterioles, leading to vasodilation and decreased blood pressure. ACE inhibitors
block the conversion of angiotensin I to angiotensin II, a potent vasoconstrictor, leading to
vasodilation and decreased blood pressure.
2. Which patient-specific factors should be considered when administering medication therapy for
hypertension?
a) Patient's weight and medical history
b) Patient's age and medical history
c) Patient's weight, medical history, and other relevant factors
d) Patient's age, medical history, and other relevant factors
Answer: c) Patient's weight, medical history, and other relevant factors
Explanation: When administering medications, it is important to consider patient-specific factors
such as age, weight, medical history, and other relevant factors. For example, in Mrs. Jones' case,
her medical history of diabetes and hyperlipidemia may impact her medication therapy for
hypertension.
Extended Drag and Drop Question: Match the drug mechanism of action to the appropriate
medication class:
Calcium channel blockers
Angiotensin-converting enzyme (ACE) inhibitors
Beta-blockers
A. Block the effects of adrenaline
B. Block the renin-angiotensin-aldosterone system, which decreases blood pressure
C. Block calcium entry into cardiac muscle cells and smooth muscle cells of the arterioles, leading
to vasodilation and decreased blood pressure
Answer:
Calcium channel blockers -C. Block calcium entry into cardiac muscle cells and smooth muscle
cells of the arterioles, leading to vasodilation and decreased blood pressure
Angiotensin-converting enzyme (ACE) inhibitors- . B. Block the renin-angiotensin-aldosterone
system, which decreases blood pressure
Beta-blockers - A. Block the effects of adrenaline

CASE: A 68-year-old male patient presents to the emergency department with complaints of chest
pain. After performing an electrocardiogram (ECG), the physician diagnoses the patient. with
acute myocardial infarction (AMI). The patient is started on aspirin and heparin and is scheduled
to undergo percutaneous coronary intervention (PCI) within the next few hours. The nurse
responsible for the patient's care should:
A) Administer an additional dose of aspirin
B) Administer an additional dose of heparin
C) Review the patient's medical history for potential contraindications to the medications
D) Administer a medication to relieve the patient's pain
Answer
C) Review the patient's medical history for potential contraindications to the medications
In this case, the nurse should review the patient's medical history to identify any potential
contraindications to the medications that have been prescribed, including aspirin and heparin.
Patients with a history of gastrointestinal bleeding or allergy to aspirin should not receive aspirin,
and patients with a history of heparin- induced thrombocytopenia should not receive heparin. The
nurse should also assess the patient for any potential adverse effects of the medications and monitor
the patient's response to the treatment.
A) Administer an additional dose of aspirin is incorrect because the patient has already been
started on aspirin, and administering an additional dose without reviewing the patient's medical
history could be harmful if there are any contraindications to aspirin.
B) Administer an additional dose of heparin is incorrect because the patient has already been
started on heparin, and administering an additional dose without reviewing the patient's medical
history could be harmful if there are any contraindications to heparin.
D) Administer a medication to relieve the patient's pain is incorrect because the patient is already
receiving medications for the treatment of AMI, and pain relief should not be the nurse's priority
in this case. The nurse should focus on ensuring that the patient is receiving appropriate
medications and monitoring the patient for potential adverse effects of the treatment.
CASE: A 45-year-old patient presents to the emergency department with symptoms of chest pain
and shortness of breath. The patient's medical history includes hypertension, hyperlipidemia, and
type 2 diabetes. The healthcare provider orders several medications for the patient, including a
beta- blocker, a statin, and an ACE inhibitor.

MCQ question:
What is the rationale for administering a beta-blocker to a patiens with hypertension and chest
pain?
A) Beta-blockers increase heart rate and blood pressure
B) Beta-blockers decrease heart rate and blood pressure
C) Beta-blockers increase insulin sensitivity
D) Beta-blockers reduce blood glucose levels
Answer: B) Beta-blockers decrease heart rate and blood pressure.
Explanation: Beta-blockers work by blocking the effects of adrenaline, which reduces heart rate
and blood pressure. They are commonly prescribed for patients with hypertension, angina, and
heart failure. In this case, the patient has hypertension and is experiencing chest pain, which may
be a sign of angina or a heart attack. Administering a beta-blocker can help reduce the patient's
heart rate and blood pressure, which can improve blood flow to the heart and reduce the risk of
further cardiac events.

CASE: John is a 70-year-old male with a history of hypertension. He was recently started on a
new blood pressure medication, amlodipine. After a week of taking the medication, he reported
constipation. He had difficulty passing stools and felt bloated. He tried increasing his fiber and
fluid intake but did not see any improvement. Which of the following statements is true regarding
John's condition?
A) John's constipation is not related to his medication.
B) Amlodipine is known to cause constipation as a side effect.
C) John needs to stop taking his medication immediately.
D) John should increase his amlodipine dosage to relieve his constipation.
Answer: B) Amlodipine is known to cause constipation as a side effect.
Explanation: Option B is correct because some medications can cause changes in bowel habits,
such as constipation, as a side effect, and amlodipine is one of them. John's symptoms are
consistent with medication-induced constipation, which can occur due to the medication's effect
on the digestive system. Options A, C, and D are incorrect because constipation is a well- known
side effect of amlodipine, and John should not stop taking his medication without consulting his
healthcare provider or increase his dosage without medical advice.

Diuretics
Diuretics are also called water pills. They help rid your body of salt (sodium) which can cause
hyponatremia as a side effect and water. Most of these medicines help your kidneys release more
sodium into your urine. This is the reason diuretics are also used in hypertension.
Safety for pregnancy period: Basically as other anti- hypertensives are contraindicated in
pregnancy, we use these water pills for pregnancy-induced and related hypertension.
We will study 4 classes of diuretics in this chapter:
 loop Diuretics,
 thiazide diuretics,
 potassium sparing diuretics and
 osmotic diuretics.
Loop diuretics are commonly used and are more efficient than other diuretics.
Before going in-depth in each of these drug classifications, please remember, some common points
which are true for all diuretics and these are quite useful for nclex examination too:
 All diuretics help in throwing out sodium and water so basically in toxic doses all diuretics
cases dehydration.
 All diuretics can induce digoxin toxicity except for potassium-sparing diuretics
 All diuretics are contraindicated in patients with lithium and this can cause lithium toxicity.
COMMON TO ALL DIURETICS
DEHYDRATION
DIGOXIN TOXICITY
LITHIUM TOXICITY

Loop diuretics
You can identify loop diuretics with the ending 'ide'
Some common examples are Furosemide, Bumetanide, and Furosemide.
These are called loop diuretics because these work on the 'Loop of Henle' in the nephron. They
inhibit the sodium and chloride reabsorption from nephron and thus impart their diuretic effect but
please keep in mind that they can alter the reabsorption of all electrolytes so you should watch
carefully the electrolyte concentration for all diuretics the magnesium, BUN, and creatinine ad uric
acid level.
As a side effect of loop diuretics such as furosemide, you may observe ototoxicity, which can result
in deafness so you should watch out for the administration rate of furosemide or other loop
diuretics, It should be infused slowly on IV.
As it can decrease electrolyte reabsorption, there could be hypomagnesemia, hypokalemia,
hyponatremia, etc. Also, remember thrombocytopenia in which there is a low platelet count which
is also a side effect of loop diuretics.
Like all other diuretics, in toxic doses, it may cause dehydration. They could induce digoxin
toxicity and are contraindicated in patients with lithium.

Thiazide Diuretics
As the name, drugs of this class of diuretics usually end with thiazide. Examples are
hydrochlorothiazide, chlorthiazide, etc.
They help in increasing sodium and water excretion. As they throw out the electrolytes or excrete
electrolytes, they may cause hyponatremia and hypokalemia and as water is also thrown out, may
cause hypotension due to hypovolemia and electrolyte imbalance.

Nursing interventions and Management for Patients on thiazide diuretics-


Check the blood sugar of patients regularly and change positions slowly to prevent orthostatic
hypotension.
All diuretic classes of drugs should better be taken in the morning as they may cause sleep
interruption as increase urine frequency.
So, what if the patient needs an immediate diuresis (for emergency diuresis)?
We should use Loop diuretics in that case for immediate diuresis
Caution with patients in lithium therapy-
They can cause digoxin toxicity, and dehydration and are contraindicated in patients undergoing
lithium therapy.
Potassium Sparing Diuretics
Potassium-sparing diuretics, help in potassium retention and promote sodium and water excretion.
The main drug for drugs of this class is Spironolactone.
Watch out for the end 'one' for drugs of this class-
Spironolactone and eplerenone etc.
other drugs are triamterene and amiloride.

Side Effects-
One common side effect could be hyperkalemia which manifests in the forms of nausea, vomiting,
weakness, diarrhea, rash, etc.
Nursing Management
What suggestion as a nurse would you give to someone with potassium-sparing diuretics?

✓ Tell him/her to avoid salty foods because they may contain potassium.
✓ Green leafy vegetables such as spinach and banana are better to avoid.

✓ Also as in severe kidney and hepatic conditions, the drug should be avoided.

✓ They can cause digoxin toxicity, and dehydration and are contraindicated in patients undergoing
lithium therapy.

Osmotic Diuretics Common examples are mannitol and isosorbide. Unlike other classes of
diuretics, they are used to decrease intra- ocular pressure. Keep keeping a note of this point. They
are usually used with chemo to induce diuresis. Drugs of this class are called osmotic diuretics as
they increase the osmolarity of blood and renal filtrate.
Side Effects:
 fluid and electrolyte imbalance
 hyponatremia
 can cause digoxin toxicity (with digoxin), and dehydration and are contraindicated in
patients undergoing lithium therapy.

Strategies for solving questions related to the diuretics class of drugs


1. Understand the mechanism of action of diuretics. Diuretics are medications that increase
the production of urine by the kidneys. There are different types of diuretics that work in
different ways. Understanding the mechanism of action of each type of diuretic can help in
answering questions related to their therapeutic effects and potential side effects.
For example, loop diuretics like furosemide work by blocking the reabsorption of sodium and
chloride in the loop of Henle in the kidneys, leading to increased excretion of water and
electrolytes. Knowing this mechanism can help in answering questions about the use of
furosemide in treating edema or hypertension.
2. Know the indications and contraindications of diuretics: Diuretics are used for different
medical conditions such as heart failure, hypertension, and edema. Knowing the indications
and contraindications of each type of diuretic can help in answering questions related to
their appropriate use.
For example, thiazide diuretics like hydrochlorothiazide are commonly used for hypertension,
but they are contraindicated in patients with renal failure. Knowing this information can help
in answering questions about the appropriate use of hydrochlorothiazide
3. Recognize potential adverse effects and drug interactions Diuretics can cause various side
effects such as electrolyte imbalances, dehydration, and hypotension. Additionally, they
can interact with other medications, such as nonsteroidal anti- inflammatory drugs
(NSAIDs) or anticoagulants. Knowing the potential adverse effects and drug interactions
of each type of diuretic can help in answering questions related to their safe administration.
For example, if a question asks about a patient who is taking furosemide and is also prescribed
an NSAID, knowing that this combination can increase the risk of renal dysfunction can help
in selecting the appropriate nursing intervention.
Example: A patient with heart failure is prescribed furosemide What electrolyte imbalances
should the nurse monitor for?
Answer: The nurse should monitor for hypokalemia, hyponatremia, and hypomagnesemia.
Furosemide is a loop diuretic that can cause the excretion of potassium, sodium, and
magnesium in the urine. Hypokalemia can cause muscle weakness, arrhythmias, and metabolic
alkalosis. Hyponatremia can cause confusion, seizures, and cerebral edema. Hypomagnesemia
can cause muscle weakness, tremors, and cardiac arrhythmias. Therefore, monitoring
electrolyte levels is important to prevent potential adverse effects of furosemide.

4. Understand the nursing implications of diuretics: Nursing implications related to diuretics


include assessing for signs of dehydration, monitoring vital signs, and monitoring
laboratory values such as electrolytes and renal function tests. Knowing the nursing
implications related to diuretics can help in answering questions related to their safe
administration and monitoring.
For example, a question may ask about the nursing intervention for a patient receiving
spironolactone, which is a potassium- sparing diuretic. Knowing that the nurse should monitor for
signs of hyperkalemia can help in selecting the appropriate nursing intervention.

5. Familiarize yourself with the brand and generic names of diuretics: Knowing the brand and
generic names of diuretics can help in identifying the correct medication and ensuring the
safe administration of medications.
For example, a question may ask about a patient who is prescribed Lasix. Knowing that Lasix
is the brand name for furosemide can help in identifying the correct medication and answering
the question correctly.
QUESTION FOR THOUGHT: A patient with hypertension is prescribed
hydrochlorothiazide. What is the appropriate nursing intervention related to this medication?
Answer: The appropriate nursing intervention related to hydrochlorothiazide is to monitor the
patient's blood pressure and electrolyte levels, particularly potassium levels.
Hydrochlorothiazide is a thiazide diuretic that can cause hypokalemia, so the nurse should
monitor the patient's potassium levels and assess for signs of hypokalemia such as muscle
weakness and cardiac arrhythmias. Additionally, the nurse should monitor the patient's blood
pressure to evaluate the effectiveness of the medication in controlling hypertension.
6. Understand the patient education related to diuretics: Patient education related to diuretics
includes information about the importance of taking the medication as prescribed, the
potential side effects, and the need to monitor for signs of dehydration. Knowing the patient
education related to diuretics can help in answering questions related to patient teaching.
For example, a question may ask about the appropriate patient teaching for a patient who is
prescribed a diuretic. Knowing that the nurse should educate the patient about the potential
side effects of the medication and the importance of monitoring for signs of dehydration can
help in selecting the appropriate nursing intervention.

7. Recognize the differences between the various types of diuretics: There are different types
of diuretics, including loop diuretics, thiazide diuretics, and potassium-sparing diuretics.
Knowing the differences between these types of diuretics, including their indications,
mechanisms of action, and potential side effects, can help in answering questions related
to the appropriate use of diuretics.
For example, a question may ask about the appropriate diuretic to use for a patient with heart
failure. Knowing that loop diuretics are often used in the treatment of heart failure due to their
potent diuretic effect and ability to decrease fluid overload can help in selecting the appropriate
nursing intervention.
Example: A patient with liver cirrhosis is prescribed spironolactone. What should the nurse
monitor for when administering this medication? Answer: The nurse should monitor for
hyperkalemia when administering spironolactone. Spironolactone is a potassium- sparing
diuretic that can cause the retention of potassium in the body, leading to hyperkalemia.
Hyperkalemia can cause muscle weakness, cardiac arrhythmias, and even cardiac arrest.
Therefore, monitoring the patient's potassium levels and assessing for signs of hyperkalemia
is important when administering spironolactone.

Digoxin
Digoxin is generally used to treat fibrillation which may be atrial fibrillation or atrial flutter and in
other episodes of heart failure and cardiogenic shocks. Though the mechanism of action is not
important just for your knowledge, it can stimulate the contraction by inhibiting the sodium-
potassium pump by acting on sodium-potassium ATPase.
So, what happens in atrial fibrillation is, there is increased conduction of frequency and high heart
rate or tachycardia. Digoxin manages both of these conditions by slowing down the heart rate and
decreasing the frequency of impulse transmission. It thus manages the heart rate and rhythm.
Side Effects:
 Digoxin slows down the heart rate right- sometimes it does that to the extent that there is
bradycardia or too slow heartbeats.
 Visual changes or disturbances- blurred vision and diplopia a term that means double image
or seeing two objects adjacently when there is only one. The patient also may suffer from
photophobia and fear to walk out in the sun.
 Other side effects are quite general headaches, weakness. and gastrointestinal effects like
vomiting, anorexia, diarrhea, and nausea).

Digoxin Toxicity
In digoxin toxicity, all these symptoms can be seen with bradycardia and its effect on vision.
Whenever the dose is greater than 5-2ng/mL, there is a risk of digoxin toxicity.
Another more important thing to memorize for your exam is the interaction of digoxin with all
other drugs and electrolytes.
If there is a low level of potassium in the blood, it can cause digoxin toxicity even in a slight dose.
Or say, conversely, if the digoxin dose is high, it can cause hypokalemia so you should expect to
see all the symptoms of hypokalemia when there is digoxin toxicity.
Not only hypokalemia but digoxin level should be well maintained for patients with
hypomagnesemia and hypercalcemia so overall, digoxin can cause electrolyte imbalance in the
blood.

Renal Clearance
Another important consideration, digoxin is renal cleared. So, it can affect the clearance of all other
drugs when given concomitantly.
So whenever you see any antibiotics given concomitantly with digoxin there are high chances that
the drugs compete with digoxin for renal clearance so the side effect of the antibiotics is more
pronounced in that case.
Talking about renal clearance, we know that aged people have problems with a renal clearance
right?
So, digoxin levels should be monitored with care for older patients too.
Making a summary out of all that we have remembered. Always remember for your exam that:
Digoxin can cause electrolyte imbalance causing various conditions like hypokalemia.
Digoxin and potassium can compete with each other.
Digoxin is excreted renal so may affect the renal clearance of many drugs and antibiotics Digoxin
dose should be monitored well for people with renal impairment electrolyte imbalance and elderly
people.
And for use, remember digoxin is used to treat heart conditions with high impulse rate such as
atrial flutter, atrial fibrillation, tachycardia, etc as it can slow down the heart rate but if not
monitored well, it can cause bradycardia.

Nitrates
Nitrates are the other important cardiovascular drugs of choice for your nclex examination.
Generally, Nitroglycerin is a popular nitrate that is used below the tongue for nitroglycerin
sublingual tablets. Another important example is isosorbide. Note that a nurse's job is to advise not
to chew the tablet it is sublingual.
It is used in myocardial infarction or heart attack as it causes vasodilation so there is more space
for blood to flow in the arteries and veins.
So, we read that nitroglycerin causes vasodilation. So, what are the side effects of vasodilation? it
could be flushing or hypotension, confusion, feeling dizzy, etc.
As a nursing intervention, your job would be to inform the patient about the side effects and
hypotension. As it causes hypotension, the patient must remain seated, the head should be lowered
down in bed. The patient must be advised not to chew the tablet that is asked to place below the
tongue or sublingually.

Diabetics Management
Diabetes is a very common disease in the US and the rest of the world so there are high chances
of questions coming in the exam on pharmacological or other management approaches to diabetes.
Without much delay let's start our first part of the series on diabetes management.

Insulin
I have divided this lecture into the time part and the nursing management approach.
A. Short or ultra-short-acting insulin
 onset time 15 minutes
 peak time = 30 minutes
 duration=3 hours.
Example: Remember LAG, Lispro (also called Humalog). Aspart, and Glulisine in this group.
Note: As this insulin begins onset in just 15 minutes and has a short duration of action, your
job is to ensure that the patient cats something within 15 minutes of drug administration, ie,
you give the drug as the patient starts to eat, not before eating.
B. Regular insulin
Basically at the end of insulins with these names we add R like humulin R and Novolin R.
onset time = 1 hour
peak time: 2 hour
duration=4 hours.
Note: This is given in the IV route.
C. Intermediate Acting Insulin
 onset time = 6 hour
 peak time 8 to 10 hours
 duration 12 hours.
Note: An example of intermediate-acting insulin is NPH. As it has 12 hours of action it is
given twice a day but remember, not to give it at bedtime as this can cause hypoglycemia at
bedtime. As a nursing intervention, you should remember that it forms a cloudy suspension
and can precipitate so it cannot be given IV as if it precipitates, it can cause under or overdose.
Long-acting Insulin
Examples: Glargine and detemir
duration= 12 to 24 hours.
Note that this is only the safest insulin for use at bedtime as it has very little chance of
sleep hypoglycemia.
Imp note for revision: Remember, NPH or intermediate insulin shouldn't be given IV as it is
in suspension form, and long-acting insulin is the safest insulin for bedtime.

Oral hypoglycemics
What are oral hypoglycemics?
Oral hypoglycemic are those drugs basically given for type 2 diabetes. In type 1 diabetes the body
produces little or no insulin so insulin therapy was needed. In type 2 diabetes, there is lesser insulin
sensitivity or lesser insulin production Basically, there is a high level of blood sugar.
All oral hypoglycemics can cause hypoglycemia so a blood sugar test at regular intervals is very
important. Also, all oral hypoglycemic causes Gl disturbances.
Biguanides (Metformin)
Metformin is a first-line drug for type 2 diabetes mellitus. This works in basically 3 ways:
 Decreasing hepatic glucose production
 Improving insulin sensitivity and
 Decreasing intestinal absorption of glucose
Side effects of metformin :
This medicine has several side effects so it is best to titer or adjust the drug dose slowly.

 diarrhea,
 Gl disturbance,
 hypoglycemia
 metallic taste in the mouth.

⚫ lactic acidosis (Also discussed in detail later-concurrent drugs such as topiramide, smokers and
people above 65 years of age are vulnerable to lactic acidosis and this can even cause death).
Note: As a mechanism of excretion of renal, we should avoid this drug in people with renal
diseases.

Lactic Acidosis
One of the main side effects is lactic acidosis. It is metabolic acidosis with signs and symptoms
such as weakness, bradycardia, hypotension, lethargy, etc., and occurs when lactic acid is built up
in the bloodstream. This also causes a metallic taste in the mouth and Gastrointestinal Upset.
Nursing interventions
 Tell the patient that alcohol shouldn't be consumed concomitantly with metformin because
it causes hypoglycemia.
 To avoid the GI effect, it shouldn't be taken on an empty stomach. That means it should be
taken with food.
 A patient may be supplemented with vitamin B12. The reason is: vitamin B12 is correlated
with vitamin B12 deficiency.
 Discontinue the drug 48 hours before any contrast test. Contrast dye such as iodine contrast
procedure for cardiac catheterization can affect the kidney function of the patient which
would be pronounced by metformin as it causes lactic acid build-up. So, a combination of
these 2 would be a disaster. This is the reason why metformin is stopped on the day or 48
hours before contrast procedures.
Sulfonylurea
Sulfonylureas end with ide.
Examples are Glipizide, Glyburide, glimepiride, Tolbutamide, Tolazamide, etc.
MOA: They work by Stimulating beta cells in the pancreas to produce more insulin.

Side Effects:
 Weight gain,
 Gl disturbances
 hypoglycemia.
 hypersensitivity reaction
 photosensitivity reaction
Allergic reactions with Sulfonylureas
A more serious side effect or say drug interaction is the cross- reaction with any drugs that contain
the sulpha moiety such as sulfa antibiotics. It can cause a serious allergic reaction to sulfonylureas.
What are sulfa antibiotics that sulfonyl ureas can have allergic or hypersensitivity reactions if given
concurrently?
sulfadiazine, sulfamethizole, sulfamethoxazole, sulfasalazine, etc.
You may have to discontinue the drug if such cases of hypersensitivity reactions appeared in
people on sulfonamide therapy.

Thiazolidinediones
Mechanisms of action:
 Increasing insulin sensitivity by acting on adipose tissue
 Increasing musculoskeletal glucose utilization or
 Decreasing glucose production in the liver
The identifier key term is the presence of-zone at the end of drugs of these classes. Examples are:
 Glitazone
 Pioglitazone
 Rosiglitazone
 Englitazone and
 Darglitazone
Note: whenever you see drugs from this class in your exam, chances are, there could be serious
side effects cases of this medication. Side Effects: o bone fractures, increased low-density
lipoproteins and o hepatotoxicity. o increased chances of myocardial infarction (should be avoided
totally for heart patients). Note: It can increase the level of low-density lipoproteins, as a nursing
intervention, we should monitor elevated levels of liver enzymes like ALT and AST for patients.

Meglitinides
Meglitinides have the ending-linide.
Examples: Repaglinide and nateglinide,
These are one of the safest drugs of this classification of oral hypoglycemic agents as they have a
short duration of action and lowest chances of lowering the blood glucose level.
Another advantage of having fast onset and short duration of action is- the client can take the drug
at the time of eating and if the patient is not having any food, the medication can be skipped.
The mechanism of action is by stimulating beta cells to produce more insulin and the side effects
are common with other oral hypoglycemic such as GI disturbances and hypoglycemia.

Alpha-glucosidase inhibitors
i. acarbose and
ii. miglitol.
MOA: They inhibit alpha-glucosidase, an enzyme that breaks down glucose for absorption. They
thus delay glucose absorption.
Side effects:
 hepatic toxicity and
 Gl effects.
Note: As alpha glucosidase drugs are hepatotoxic, liver function tests should be regularly done
and old and patients with hepatic diseases should not be given alpha-glucosidase inhibitors.
Strategies for solving insulin and antidiabetic medication related problems for nclex
1. Understand the mechanism of action: Understanding how a drug works in the body is
essential to answer medication administration questions.
For example, insulin is a hormone that is secreted by the pancreas to regulate blood glucose
levels. Insulin works by stimulating glucose uptake into cells, converting glucose to glycogen,
and inhibiting the release of glucose from the liver. If a patient has diabetes, they may not
produce enough insulin, and insulin injections may be necessary to manage their blood glucose
levels.
Example: A patient with type 1 diabetes requires insulin injections to manage their blood
glucose levels. Which of the following statements about insulin is true?
a) Insulin inhibits glucose uptake into cells
b) Insulin converts glycogen to glucose
c) Insulin stimulates the release of glucose from the liver
d) Insulin is a hormone that regulates blood glucose levels
Answer: d) Insulin is a hormone that regulates blood glucose levels.
2. Know the side effects: Understanding the side effects of a drug is crucial to answer
pharmacology questions. For example, antidiabetic drugs like metformin can cause
gastrointestinal side effects like diarrhea, nausea, and abdominal pain.
Example: A patient is taking metformin for type 2 diabetes. Which of the following side effects
is commonly associated with this medication?
a) Bradycardia
b) Hypertension
c) Diarrhea
d) Insomnia
Answer: c) Diarrhea.
3. Consider contraindications: Understanding the contraindications of a drug is important to
answer medication administration questions. For example, sulfonylureas are
contraindicated in patients with type 1 diabetes.
Example: A patient with type I diabetes is prescribed a sulfonylurea. Is this medication
appropriate for this patient?
a) Yes, sulfonylureas are commonly prescribed for type 1 diabetes
b) No, sulfonylureas are contraindicated in type 1 diabetes
c) It depends on the patient's blood glucose levels
Answer: b) No, sulfonylureas are contraindicated in type I diabetes.
4. Considering the patient's lifestyle and individualized treatment goals: The nurse should
consider the patient's lifestyle and individualized treatment goals when solving insulin and
antidiabetic medication-related problems. For example, if a patient is having difficulty
adhering to a strict insulin regimen due to work or family obligations, the nurse may need
to work with the patient to develop a more flexible insulin regimen that fits the patient's
lifestyle.
Example: A patient with type 2 diabetes is prescribed metformin. The nurse should educate the
patient on the importance of taking the medication with meals to reduce the risk of
gastrointestinal side effects, such as diarrhea. The nurse should also monitor the patient's renal
function, as metformin can cause renal impairment in some patients.

5. Monitoring blood glucose levels regularly: Regular monitoring of blood glucose levels is
important to identify and address any medication-related problems promptly. For example,
if a patient's blood glucose levels are consistently high despite adherence to an insulin
regimen, the nurse may need to adjust the insulin dosage or consider adding a different
type of medication to the treatment plan.
Example: A patient with type 1 diabetes is prescribed a basal- bolus insulin regimen. The nurse
should instruct the patient on the importance of checking their blood glucose levels before
meals and at bedtime to ensure that their insulin dosage is adjusted appropriately. The nurse
should also educate the patient on the signs and symptoms of hyperglycemia and the
importance of contacting their healthcare provider if their blood glucose levels are consistently
high.

Proton Pump Inhibitors


One of the most commonly prescribed drug class are the Proton pump inhibitors (PPIs) These are
used for alleviating the conditions such as heartburn and acid-related disorders. They work by
blocking the site of acid production in the parietal cell of the stomach.
So, these can be used for heartburn, to Relieve symptoms of acid reflux, gastroesophageal reflux
disease (GERD) and also for duodenal or stomach ulcers.
Just remember that PPIs shut down pumps in the stomach that produce excess acid. For your nclex
preparation, these are the drugs that end with 'zole' or 'prazole'.
Examples are:
 Omeprazole
 Esomeprazole
 Lansoprazole
 Rabeprazole
 Pantoprazole
 Dexlansoprazole
So, let us remember some points which could be quite helpful for your nclex preparation.
 Proton Pump Inhibitors are usually given 30 minutes before meals (on an empty stomach)
as this ensures enough time for activation, absorption, and therapeutic action. It is not safe
even for a child. What are safe PPI for the child then? Your answer should be lansoprazole
 Let's talk about drug interaction through which some of the questions may fall in the exam.
For patients undergoing Benzodypines, phenytoin, and warfarin, there is a chance of
increased serum levels in proton pump inhibitors being given concomitantly so we should
closely observe the blood level of these drugs.
 Avoid this combination- Clopidogrel with proton pump inhibitors because there are
increased cardiovascular risks with proton pump inhibitors.
 Also remember, Long term use of PPIs affects intestinal magnesium absorption leading to
hypomagnesemia.
 Another thing to memorize is proton pump inhibitors are enteric-coated. That's a special
coating of a drug as these are acid-labile, and require an enteric coating to protect them
from degradation in the stomach when given orally.
You learned about the properties of
 Omeprazole
 Esomeprazole
 Lansoprazole
 Rabeprazole
 Pantoprazole
 Dexlansoprazole

Remember 'prazole' or 'zole' at the end when you think of proton pump inhibitors.

CASE: Mary is a 60-year-old patient with a history of heartburn and acid reflux. Her healthcare
provider prescribes a proton pump inhibitor (PPI) to manage her symptoms. Mary is confused
about when to take the medication and asks the nurse for guidance. Which of the following
statements made by the nurse is correct?
A) "Take the PPI with meals to avoid stomach upset."
B) "It is safe to give PPIs to children."
C) "PPIs can be taken with benzodiazepines and warfarin without any concerns."
D) "Clopidogrel should not be taken with PPIs due to increased cardiovascular risks."
E) "Long-term use of PPIs may increase intestinal magnesium absorption."
F) "Proton pump inhibitors are not enteric coated."
Answer
The correct answer to the case study is A) "Take the PPI 30 minutes before meals (on an empty
stomach) as this ensures enough time for activation, absorption, and therapeutic action. It is not
safe even for a child. What are safe PPIs for a child then? Your answer should be lansoprazole."
Explanation: PPIs should be taken on an empty stomach, 30 minutes before meals, to ensure
sufficient time for activation and absorption. PPIs are not safe for children, except for lansoprazole,
which is the only PPI that has been approved for use in children.
Which of the following statements about proton pump inhibitors (PPIs) are true? Select all that
apply.
A) PPIs are used to treat heartburn and acid-related disorders.
B) PPIs work by blocking the site of acid production in the stomach.
C) PPIs should be taken with meals to avoid stomach upset.
D) It is safe to give PPIs to children.
E) PPIs can increase serum levels of benzodiazepines, phenytoin, and warfarin when given
concurrently.
F) Long-term use of PPIs may decrease intestinal magnesium absorption.
The correct answers to the MCQ question are A) PPIs are used to treat heartburn and acid-related
disorders, B) PPIs work by blocking the site of acid production in the stomach, and E) PPIs can
increase serum levels of benzodiazepines, phenytoin, and warfarin when given concurrently.
Explanation: PPIs are used to treat heartburn and acid-related disorders, and they work by
blocking the site of acid production in the parietal cell of the stomach. PPIs can increase serum
levels of benzodiazepines, phenytoin, and warfarin when given concurrently, which can lead to
potential drug interactions.
The other options, C) PPIs should be taken with meals to avoid stomach upset, D) It is safe to give
PPIs to children, F) Long- term use of PPIs may decrease intestinal magnesium absorption, are all
incorrect statements. PPIs should not be taken with meals, except for Dexlansoprazole, which can
be taken with or without food. PPIs are not safe for children, except for lansoprazole. Long-term
use of PPIs may increase intestinal magnesium absorption, not decrease it.

Respiratory medicines
Welcome to the new lecture series on medicines used to treat diseases of the respiratory system.
In this lecture series, we will study the drugs, side effects and nursing management of the diseases
and caring clients with respiratory diseases.

Asthma
Asthma is a chronic airway inflammatory disease with hypersensitiveness to various stimuli such
as environmental stimuli (dust, pollen, smoke, etc), drugs stimuli such as beta blockers and aids,
physiological stimuli such as stress, GERDS, sinusitis, etc or any other food and occupational
factors.

Nursing considerations
 See for the following symptoms while assessing this disease:
 See for cough and wheezing.
 See for dyspnea. Dyspnea means an intense tightening in the chest, air hunger, difficulty
breathing, breathlessness, or a feeling of suffocation.
 See for wheezing. Wheezing means the sound or whistle you hear when your airway is
partially blocked. Generally in asthma expiration needs an effort and sound is produced.
 There is also decreased oxygen saturation and as a result cyanosis and diaphoresis.

Status asthmaticus
When the condition is more severe, we give it the term status asthmaticus. If an attack comes on
quickly and it doesn't respond to regular treatment, it can lead to status asthmaticus,
This condition is unresponsive to drug therapy such as inhaled albuterol, levalbuterol, or
subcutaneous epinephrine. It is a medical emergency that requires immediate recognition and
treatment.
So, what could be the nursing management for asthma?
We should put the patient in Fowler's position. Fowler's position is a standard patient position in
which the patient is seated in a semi-sitting position (making an angle of 45-60 degrees with the
floor) and may have knees either bent or straight.
We then administer oxygen and bronchodilators or corticosteroids to the patient as prescribed and
then to see the progress, we auscultate the lung sounds.

Beta 2 adrenergic agonist- Bronchodilators


Bronchodilators are the agents that dilate the mucous membrane or smooth muscle of the bronchi.
In doing so, the surface area for air exchange increases so breathing and air exchange becomes
casy.
So, how do you identify Beta 2 adrenergic agonist bronchodilators?
You have to identify the drugs ending with 'terol' or 'ol'
Example are:
 Albuterol
 Levalbuterol
These both are short-acting bronchodilators.
They work within 15 minutes and are an efficient choice for new asthma conditions.
Similarly, we also have long-acting bronchodilators.
Example are:
 Salmeterol .
 Formoterol
Long-acting bronchodilators are available in dry powder inhaler and nebulizer form. They act for
as long as 12 hours. Note that both of these drugs are beta 2 agonists so they show
sympathomimetics properties.
So, what are the sympathomimetic side effects?
These include
 Nervous or shaky feeling.
 Mouth Dryness.
 Overexcitement or hyperactivity and hypertension
 Increased heart rate or Tachycardia and Dysrhythmias
 Hyperglycemia
These Bronchodilators are used in combination with ipratropium and are an excellent choice for
acute asthma attacks.

Methylxanthines
These are also kinds of bronchodilators and end with phylline They stimulate the respiratory
centers to dilate the coronary and pulmonary vessels and thus relaxation of the smooth muscles.
common examples are theophylline which is given in an oral route and Aminophylline.
So, how do they work?
Methylxanthine relaxes the smooth muscles located in the bronchial airways and pulmonary blood
vessels. Note that, Theophylline is also used to treat the symptoms of other lung conditions that
block your airways, such as emphysema or chronic bronchitis. It's used for long-term treatment.
Side Effects
We will be discussing the side effects of one of the popular drugs widely used in asthma, that is
theophylline.
We will be learning side effects and nursing management principles to manage those symptoms
side by side.
First of all, it can cause gastric irritation. As a nursing management approach, you would give this
medicine only with food and educate the client not to take the medicine on an empty stomach.
Second, like all other respiratory medicines, these are sympathomimetics. They show side effects
like Tachycardia, Insomnia, restlessness, confusion, Dysrhythmias, etc. You should educate the
patient that these side effects are normal.
Third, we learned that beta 2 agonists like albuterol and salmeterol cause Dysrhythmias. This class
of drug if used with beta 2 agonists can also cause dysrhythmias.
If you are new to this term cardiac dysrhythmia means imple or abnormal heartbeat. It could be a
marker of heart disease.
Another thing to watch out for is regarding correct dose and toxicity. The therapeutic dose of
theophylline is 10 to 20 mcg/ml. Do not let the serum level by more than 20 mcginl because this
can lead to theophylline toxicity. You can access toxicity with severe sympathomimetic signs such
as tachycardia, restlessness, tremor, and palpitations.
Another important point is a drug interaction. This drug should not be given with antibiotics like
ciprofloxacin or cimetidine because they disturb the absorption of theophylline so the blood level
of the drug can dramatically increase and this may cause theophylline toxicity.
It is available in enteric-coated tablet form or sustained-release tablets or capsules. Enteric coating
helps to protect this drug from the acidic environment of the stomach so that it can be absorbed
well from the lower GIT tracks.
If used IV, the drug (theophylline) should be administered slowly via the infusion pump. The drug
should be given at regular intervals so that a normal therapeutic level is achieved. Patients
undergoing the drug therapy should not be taking any cigarettes, tea, coffee, chocolates, etc
because such products worsen respiratory conditions. This point is common for all other drugs
used to treat asthma or other respiratory diseases.

Glucocorticoids
Glucocorticoids are another class of drugs used to treat restrictive airway disorders like asthma
and COPD.
Identify the drugs falling in this class with the ending-sone or – ide
Examples are beclomethasone, prednisone, fluticasone, mometasone furoate, Budesonide,
ciclesonide, triamcinolone, etc.
One important property of drugs of this class is- they are used for long-term treatment of airway
restriction diseases like asthma and are never used for short acute and emergency usage.
Like other respiratory medicines, also show sympathomimetic activities and side effects. They
block the muscarinic receptors in bronchioles and thus cause bronchodilation.

Side Effects
As mentioned before, Glucocorticoids show sympathomimetic properties. Examples of
sympathomimetic side effects are:
 Palpitations, tachycardia and hypertension,
 Constipation,
 Blurred vision
 Dry mouth,
 Urinary retention, and drying of body secretions.

One of the most important side effects of glucocorticoids is immunosuppression. It is for this
reason, that the blood profile is regularly seen or tested for signs of infection like the elevation of
the white blood cells. Generally, infections of the mouth are common so you should instruct the
patient to take good care of oral health.
So, this was all about the drug Glucocorticoids used to treat asthma and other disorders of the
respiratory system that causes air restriction.

Antihistamines
Antihistamines or HI blockers compete with histamine receptors to block any effects that histamine
brings into the body.
Examples are:
Diphenhydramine, Cetirizine, levocetirizine, loratadine, olopatadine, brompheniramine,
chlorpheniramine, etc.
As this drug blocks the effect of histamine, there is relaxation of smooth muscles of the smooth
muscles lining the nasal cavity and a decrease of nasal secretions, they can also be used to prevent
diseases that can cause histamine overactivity like nausea, vomiting, common cold, rhinitis, etc.
As with all respiratory drugs they too have sympathomimetic effects.
Examples are:
 Palpitations,
 tachycardia
 hypertension,
 Constipation,
 Blurred vision
 Dry mouth,
 Urinary retention, and drying of body secretions.

Let's see the nursing interventions.

 Instruct the patient not to take the drug with alcohol, opioids, or any tranquilizers because
they can cause CNS depression.
 As we studied they dry out the mucus membrane or any other body secretions so
antihistamines should not be the drug of choice for COPD patients.
 Drugs like cimetidine or loratadine are frequently used to treat motion sickness. If used for
motion sickness it should be used 30 minutes before traveling.
 Another drug of this class, diphenhydramine has anticholinergic effects so this drug is
contraindicated for glaucoma.

Leukotriene modifiers
Another class of drugs used to treat asthma is leukotriene modifiers. Note that drugs of this class
are used to treat chronic asthma conditions only so should not be the drug of choice for acute
asthma attacks.
See for the ending-lukast to identify these drugs
Examples are: Monteleukast and Zafirlukast
 Like other respiratory medicines used to treat asthma, they too relax the smooth muscles
of bronchioles and the respiratory tract. .
 This drug is cleared through the liver so care should be taken with clients with hepatic
disorders. Liver function tests should be done to access the liver enzymes like AST and
ALT.
 They can cause cyanosis as a side effect. The therapeutic dose should always be maintained
and the drug shouldn't be discontinued even if there are no symptoms unless the health care
worker prescribes to do so.
 These drugs shouldn't be used by pregnant and breastfeeding mothers and hypersensitive
individuals.
 This drug can cause immunosuppression if used for a long time so a blood profile or WBC
profile check should be done.
 We should never administer the glucocorticoids such as prednisone, beclomethasone, etc
with Leukotriene modifiers because they can increase the chances of upper respiratory tract
infections.

Nasal decongestants.
Note that nasal decongestants are also used as a treatment of choice for hay fever, allergic rhinitis,
coryza, and asthma.
Examples are:
naphazoline, xylometazoline, oxymetazoline, tetrahydrozoline, etc ending with the term-zoline at
the end, and also drugs like Phenylephrine hydrochloride and pseudoephedrine hydrochloride.
So, what could be the side effects, contraindications, and nursing management for patients who
are taking nasal decongestants?
 As with all other respiratory drugs, these drugs have sympathomimetic activity. So, watch
out for symptoms like Hypertension, Hyperglycemia, palpitations and restlessness,
insomnia, etc.
 These drugs if used for more than 2 days can cause rebound nasal congestion due to
irritation of nasal mucosa so care should be taken to take drugs for more than 2 days.
 These drugs can cause dysrhythmia and also should not be used by hypertensive patients
and patients with other cardiac diseases.
 As this drug can cause hyperglycemia due to sympathomimetic effects, these should not be
used by patients with diabetes mellitus.

Opioid Antagonists
Opioid antagonists are those drugs that can reverse respiratory depression, particularly in opioid
overdose.
Examples of Opioids are Codeine sulfate, Hydrocodone, and Codeine phosphate.
Examples of opioid antagonists are Naloxone and Naltrexone
With these drugs too the side effects are common as with other respiratory or anti-asthma drugs
such as tachycardia and palpitations, Hypertension, restlessness, tremor, vomiting. sweating, etc.
Note that although drugs of this class are used to treat respiratory depression, if the duration of
opioids given is more than the duration of administration of these anti-opioids.
As these drugs can cause severe sympathetic effects, you should have all the resuscitative medical
equipment and oxygen available if this drug is being given in IV.
Also, note one thing the drugs of this class are used to treat opioid-related respiratory depression
so shouldn't be used to reverse the respiratory depression of non-opioid antitussives such as
Dextromethorphan and Diphenhydramine.

Strategies to solve 'respiratory medications' related questio for next generation


nclex:
1. Understand the mechanism of action and therapeutic uses of respiratory medications:
Understanding the mechanism of action of respiratory medications will help you determine
how they work and their therapeutic can also understand the possible side effects and
interactions of these medications.
Example: Bronchodilators, such as albuterol, work by relaxing the smooth muscles of the airways
and increasing airflow to the lungs. These medications are used to treat asthma, chronic obstructive
pulmonary disease (COPD), and other respiratory conditions.
2. Learn the dosage and administration of respiratory medications: Learning the correct
dosage and administration of respiratory medications is crucial to ensure their efficacy and
prevent adverse effects.
Example: Inhaled medications, such as metered-dose inhalers (MDIs) and nebulizers, are
commonly used to deliver respiratory medications. When administering an MDI, the patient must
be instructed to take a slow, deep breath and hold it for 5-10 seconds after inhaling the medication.
In contrast, when using a nebulizer, the medication is delivered in a fine mist that the patient inhales
over several minutes.
3. Understand the potential adverse effects of respiratory medications: Understanding the
potential adverse effects of respiratory medications will help you identify adverse reactions
and respond appropriately.
Example: Corticosteroids, such as prednisone, are commonly used to treat inflammation in the
airways. However, these medications can cause adverse effects such as hyperglycemia, weight
gain, and immunosuppression. Healthcare providers must monitor patients taking corticosteroids
for these potential adverse effects.
4. Know the patient education points related to respiratory medications: Patients need to know
how to use respiratory medications effectively and safely, and understanding patient
education points related to respiratory medications is essential.
Example: Patients using inhaled medications need to know how to use their inhalers correctly and
clean them regularly to prevent infection. They also need to know when to seek medical attention
if their symptoms worsen or if they experience adverse effects from their medications.

5. Understand the different classes of respiratory medications. Respiratory medications can


be classified into different groups based on their therapeutic uses, such as bronchodilators,
corticosteroids, leukotriene modifiers, and mast cell stabilizers. Understanding the
different classes of respiratory medications can help you identify which medication is
appropriate for a specific patient and condition.
Example: Mast cell stabilizers, such as cromolyn sodium, work by preventing the release of
histamine and other inflammatory mediators from mast cells. These medications are used to treat
allergic asthma and exercise-induced asthma.
6. Be aware of drug interactions with respiratory medications: Some medications can interact
with respiratory medications and alter their efficacy or cause adverse effects. Being aware
of drug interactions can help you avoid potential medication errors and ensure patient
safety.
Example: Beta-blockers, commonly used to treat hypertension, can reduce the effectiveness of
bronchodilators and worsen asthma symptoms. Healthcare providers need to monitor patients
taking beta-blockers and adjust their respiratory medications as necessary.

7. Understand the importance of medication reconciliation: Medication reconciliation is the


process of comparing a patient's current medications with their medical records to avoid
medication errors and adverse drug events. Understanding the importance of medication
reconciliation can help you ensure patient safety and prevent medication- related errors.
Example: A patient may be taking multiple respiratory medications for different conditions, such
as asthma and COPD. During medication reconciliation, the healthcare provider needs to ensure
that the patient is taking the correct dosage of each medication and that there are no potential drug
interactions.
8. Be familiar with the different routes of administration for respiratory medications:
Respiratory medications can be administered via different routes, including oral, inhaled,
and intravenous. Being familiar with the different routes of administration can help you
understand the pharmacokinetics and pharmacodynamics of these medications.
Example: Oral corticosteroids, such as prednisone, are used to treat severe asthma exacerbations.
These medications are absorbed in the gastrointestinal tract and have a slower onset of action
compared to inhaled corticosteroids.
9. Understand the different delivery devices for inhaled respiratory medications: Inhaled
respiratory medications can be delivered via different devices, such as metered-dose
inhalers (MDIs), dry powder inhalers (DPIs), and nebulizers. Understanding the different
delivery devices can help you instruct patients on proper inhalation techniques and ensure
accurate medication administration.
Example: DPIs are commonly used to deliver bronchodilators and corticosteroids. These devices
do not require a propellant and deliver medication as a dry powder that the patient inhales. Patients
need to be instructed to breathe in quickly and deeply to ensure effective medication delivery.
10. Be familiar with the different formulations of respiratory medications: Respiratory
medications can be formulated as liquids, powders, and aerosols. Being familiar with the
different formulations can help you determine the appropriate medication for a specific
patient and condition.
Example: Antibiotics, such as azithromycin, can be formulated as a liquid suspension for oral
administration or as an aerosol for inhalation. The aerosol formulation is used to treat bacterial
infections in the lungs, such as pneumonia.

Anti-tuberculosis drugs
When it comes to the anti-tubercular drugs, you see that generally first line anti TB drugs are more
common. Basically, there are two classes o medications for tuberculosis, first line and second line.
What are first-line drugs?
These are:
 Isoniazid,
 Rifampin,
 Ethambutol and
 Pyrazinamide.
We also have other drugs that perhaps you must have heard of and used for curing other diseases
and which can also be used as anti-tuberculosis drugs. These are called second-line drugs for curing
tuberculosis.
Some of the common second-line drugs are:
 Rifabutin,
 Rifapentine,
 Streptomycin,
 Levofloxacin,
 Amikacin,
 Moxifloxacin etc.
Isoniazid This is bactericidal meaning it kills bacteria by interfering with DNA metabolism and
is generally always used in combination with other anti-TB drugs. It is used also in cases of latent
TB where all the symptoms have not yet come to the surface of clinical evaluation.
Side effects:
 nephrotoxicity,
 peripheral neuropathy
 hepatotoxicity
 ototoxicity, etc

As nursing considerations make sure you educate the patient not to take this drug with antacids
and foods as they may cause a delay in absorption. Also, we studied that it has chances of
neurotoxicity, and to eliminate this side effect, make sure that pyridoxine or vitamin B6 is
prescribed to the patient.
As this drug is hepatotoxic you should also make sure that liver function tests are properly accessed
before and during the administration of isoniazid to the patients.

RIFAMPIM
Rifampin inhibits RNA synthesis of bacteria
Let's study 3 important points with Rifampin,
i. As with all other anti-tb drugs, this drug to is hepatotoxic so this should be used very
wisely for alcoholic patients as the risk for hepatotoxicity increases more in alcoholic
patients.
ii. Rifampin has the property of coloring body parts yellowish by rifampin. The patient's
urine and body secretions such as sweat and tears become yellowish. Tell the patient
that these are all normal side-effect.
iii. Drug-drug interaction-
✓ Do not give these drugs to patients already on digoxin.
✓ Use care with patients' anticoagulants, rifampin may decrease the effect of
anticoagulants, and further, take good care of people with diabetes and in oral
hypoglycemic therapy.
✓ For other drugs such as fluconazole, anti-hypertensives like verapamil, methadone,
and also phenytoin. The disease profile and concomitant use of any of these drugs
should be properly accessed for patients who are going to be prescribed rifampin.
✓ Watch out for any hypersensitive reactions which may come in form of fever and
chills, muscle pain, headache, etc.
✓ You should access if the patient shows any hypersensitivity reactions and if seen, the
drug has to be immediately stopped.

Ethambutol
Ethambutol is another drug in this class though being slowest in acting so should be used in
combination with other anti- tuberculosis drugs.
Important points for other nclex examinations:

✓ First, this drug can aggravate a condition called optic neuritis. There could be ocular
inflammation with this drug.

✔Second, as with other drugs of anti tb first-line drugs, there are chances of hypersensitivity
reactions and neurotoxicity which shows the signs of tingling sensations in the hands and feet.
Also, there could be other adverse reactions such as Nausea, vomiting, Anorexia, etc and you
should understand that these side effects are common for other anti-tuberculosis drugs also.

Pyrazinamide
The important points for this drug are
It can cause cross-sensitivity with other anti-tuberculosis drugs so this is the main point you have
to watch out for. Generally, this can cause cross-sensitivity reactions with isoniazid
It should be very wisely used for patients with gout, and hepatic disorders as it can increase uric
acid levels or can increase hepatic enzymes, also should be used with care for diabetic patients, as
there are chances of hypersensitivity reactions with this drug.
It can cause photosensitive reactions so patients must be warned to avoid walking in the sunlight
without protective clothes or UV protectors applied.
Drug-drug interactions-This drug can decrease the effect of probenecid and allopurinol and many
other drugs so the concomitant other drugs use should be properly accessed.
Note for anti-tb drugs: Anti-tb drugs are used in many combinations and treatment can go from
6 months to a year. There is a chance of drug resistance if any other of these drugs is discontinued
in the middle of the term so the patient should be advised to remain stuck with the usual drug
routine.
Also, note, anti-tb drugs like rifamycin or rifampin can decrease the efficacy of these oral
contraceptives.
Strategies for solving Tuberculosis related drugs related questions for next
generation nclex
1. Understand the mechanism of action and indications for each drug: Understanding how
each drug works and its indications will help you identify the correct drug to use in a given
situation.
For example, rifampin is a first-line drug used to treat pulmonary TB. Its mechanism of action is
to inhibit RNA synthesis, making it effective against the mycobacterium that causes TB.
2. Memorize the adverse effects and interactions of each drug: Knowing the potential side
effects and drug interactions will help you identify and manage adverse reactions to
medications.
For example, a common adverse effect of isoniazid, another first-line TB drug, is peripheral
neuropathy, which can be managed by giving the patient vitamin B6 supplements.
3. Understand the nursing implications for each drug Knowing the nursing implications for
each drug will help you administer medications safely and effectively.
For example, ethambutol, another first-line TB drug, can cause optic neuritis, so nurses should
assess the patient's vision before and during treatment.

4. Understand the pharmacokinetics of each drug: Knowing the pharmacokinetics of each drug,
including its absorption, distribution, metabolism, and excretion, will help you understand how the
drug works in the body. For example, isoniazid is rapidly absorbed and metabolized by the liver.
5. Familiarize yourself with the monitoring parameters for each drug: Knowing the monitoring
parameters for each drug will help you identify when a medication is effective or when a dose
adjustment is necessary.
For example, a patient taking rifampin should have regular liver function tests to monitor for
hepatotoxicity.
6. Understand the patient education and counseling points for each drug: Educating patients about
their medications will help them adhere to their treatment regimen and avoid potential adverse
effects. For example, patients taking TB medications should be counseled on the importance of
taking their medications as prescribed and the potential for drug interactions.
7. Understand the common adverse effects of each drug: Knowing the common adverse effects of
each drug will help you recognize and manage adverse reactions.
For example, ethambutol can cause optic neuritis, which can lead to visual impairment.
8. Understand the potential drug interactions of each drug: Knowing the potential drug interactions
of each drug will help you identify situations where a medication should be avoided or used with
caution.
For example, rifampin can decrease the effectiveness of oral contraceptives.
9.Practice calculation of drug dosages and rates: Calculating drug dosages and rates is essential in
medication administration.
For example, if a patient needs to receive 300mg of Isoniazid daily and the medication comes in
100mg tablets, the patient would need to take three tablets per day.
Lithium (Mood Stabilizer)
Lithium is an antipsychotic drug. Remember the term Bipolar disorder when we talk about
lithium. It is used to treat or control the manic episodes of bipolar disorder.
Important things to understand for your nclex examination:
 Remember, lithium is teratogenic. Teratogens are agents that can cause disturbance in the
growth of the growing fetus or baby.
 Never, administer lithum to a pregnant woman.
 Lithium can cause hyponatremia or a serious lack of sodium in the body. And in this
response, the body tries to throw out water to maintain the balance of osmotic
equilibrium. So as an important nursing consideration please remember, you should not
be giving any forms of diuretics, be it osmotic diuretics or any loop diuretics to a patient
under lithium therapy because it can cause serious hypovolemia and dehydration.
 Conditions such as dehydration and diuretics such as coffee and tea can cause lithium
toxicity. So, you should advise the patient to take plenty of water to cure dehydration
effects. As it can cause hyponatremia, you can take the patient take electrolytes as well.
 You should also advise the caretakers of patients that side effects like numbness and
tingling in the skin are just symptomatic of side effects of lithium and there could be other
side effects like peeing and pooping or the patient feeling slow, lethargic, and drowsy.

Antipsychotics
There are two types of antipsychotics
Typical: Drugs like Haloperidol and Chlorpromazine fall in this group. These classes of drugs
show extrapyramidal symptoms like inability to sit still, involuntary muscle contraction, tremors,
stiff muscles or muscles rigidity, and involuntary facial movements.
Atypical: Atypical psychotics are also called "second-generation antipsychotics." They are newer,
and they tend to be used more commonly nowadays because they are less likely to cause side
effects. Drugs like Olanzapine, Quetiapine, Risperidone, etc fall in this class.
So when we go for typical or atypical antipsychotics is a question of prime importance.
For lesser side effects we go generally for second class, which is Atypical antipsychotics but still,
these are the drugs of choice when there are psychotic symptoms like hallucinations or delusions.
On the other hand, Atypical antipsychotics are used if there are not these symptoms present but
other milder psychotic symptoms like catatonia and anhedonia. Catatonia involves a lack of
movement and communication, and also can include agitation, confusion, and restlessness and
anhedonia means the inability to feel pleasure in normally pleasurable activities.
Side effects:
Extrapyramidal effects are side effects such as the inability to sit still, involuntary muscle
contraction, tremors, stiff muscles or muscle rigidity, involuntary facial movements, etc. Other side
effects may be blurring of vision, drowsiness, and effects like constipation, urinary retention, dry
mouth, and blurred vision which are all anticholinergic.
Nursing interventions and management
Any drugs in this family can cause gastric irritation so these drugs should be given after food. Also,
these drugs should be protected from light and often kept in amber-colored vessels.
Remember an important drug Haldol or Haloperidol. Haloperidol is a medication that works in the
brain to treat schizophrenia. It is also known as a first-generation antipsychotic (FGA) or typical
antipsychotic.
You may need to suggest to the patient’s caretakers that there may be a change of urine to rediism,
pinkish or reddish brown which is harmless.
Haldol is very popular for its EKG interpretation called Torsades de pointes. Torsades de pointes
is a specific form of polymorphic ventricular tachycardia in patients with a long QT interval. It is
characterized by rapid, irregular QRS complexes, which appear to be twisting around the
electrocardiogram (ECG) baseline. This is very fatal if not treated properly.

Tri-cyclic antidepressants
When we are talking about psychiatric medication, antidepressants and mainly tricyclic
antidepressants are major questions of choice. We all know that neurons talk with each other with
the help of neurotransmitters to transmit the impulse or say the message from one part of the body
to the brain and vice-versa. So, these impulses are transmitted with the help of neurotransmitters
like norepinephrine and serotonin.
They are so-called because there are three rings in the chemical structure of these medications.
Tricyclic antidepressants block the reuptake of norepinephrine and serotonin.
We identify these classes of drugs with the ending of "triptyline".
Examples are: Amitriptyline and Nortryptyline
These classes of drugs have anticholinergic effects.
Examples of anticholinergic effects are:
 constipation,
 urinary retention,
 dry mouth and eyes,
 blurred vision,
 increased heart rate, etc.
Nursing care and management.
 Note the antidote for an overdose of tricyclic antidepressants. It is Physostigmine,
Physostigmine is the Antidote for Anticholinergic Syndrome.
 It is also used for the treatment of glaucoma.
 As a nurse, you should have access to read the patient's EKG during the treatment.

An ‐seizure medicines.
The main anti-seizure medications are:
 Carbamazepine.
 Phenytoin,
 Lorazepam,
 Gabapentin
 Pregabalin
 Phenobarbital and
 Clonazepam, etc
Let's only focus on the main pharmacological points of these drugs which could be important for
your nclex examination.
These drugs should be used with care or continuously monitored for patients who are on
antipsychotic or anti coagulants medications like warfarin already as they may cause medical
interaction.
Some supplements, milk, and antacids can interfere with its absorption so your job will be to
instruct the patient not to take these drugs or supplements or antacids when the anti-seizure
medicines are being given.

Phenytoin
Let's talk in brief about one of the drugs called phenytoin as it is more of a topic of choice in the
examination.
Note that phenytoin is used to treat tonic-clonic and partial seizures.
There could be various side effects of this medication. Some examples are:
 Swelling and bleeding of the gums
 Headache,
 nausea,
 vomiting.
 constipation, dizziness
 feeling of spinning,
 drowsiness,
 trouble sleeping,
 nervousness, etc.

Phenytoin
We studied that one of the side effects of antiseizure drugs like phenytoin is bleeding so the gums
so we should instruct the patient to take good care of oral hygiene and oral health.
The drug phenytoin can increase the metabolism of estrogen which is contained in birth control
pills. So, it can decrease the efficiency of birth control pills. So, birth control pills cannot rely on
as a family planning medication if the woman is taking phenytoin.
Next, this drug is also a category D medication which may impart harm to the growing fetus so
far as possible it is best to avoid using the drug according to the prescription and discussion with
a health care worker if the woman is pregnant.
Similarly, as we talked about before, we should instruct the patient not to give these drugs
concomitantly with milk and antacids or any calcium supplement as they may delay the absorption
of these drugs.
Some of the anti-seizure medicines may cause drowsiness or dizziness or even visual impairment
disturbances so we may advise the patient to be more alert and conscious if he or she is already in
the anti-seizure medical therapy.
And if there are excessive and frequent symptoms of side effects, including bleeding from the
gums, Headache, nausea, vomiting, constipation, dizziness, the feeling of spinning, drowsiness,
trouble sleeping, nervousness, etc, the patient may need to refer to the doctor.

Antianxiety drugs
Antianxiety drugs are drugs used to treat anxiety.
Watch for the ending with - zepam and zolam for identifying antianxiety drugs.
Examples are:
Diazepam (Valium)
clonazepam
oxazepam
Alprazolam
Lorazepam
Midazolam etc.
The side effects of these drugs are similar to that of tricyclic antidepressants. They too show
anticholinergic effects. Examples of anticholinergic effects are:
 constipation,
 urinary retention,
 dry mouth,
 dry eyes,
 blurred vision and increased heart rate, etc
 They also cause drowsiness so as a nurse, you should be advising the patient not to take
drugs when the patient is driving or in any situation that demands attention.
 It is contraindicated in glaucoma. So what happens in glaucoma is there is an increase in
intraocular pressure with diazepam or any benzodiazepines.
 Benzodiazepines affect papillae muscles, their use could be a risk factor for acute angle-
closure glaucoma.
Note: The antidote for an overdose of benzodiazepine is Flumazenil. It reverses back the sedation
effects so note that this drug is also used in the reversal of the symptoms in general anesthesia.
Guess what, benzodiazepines work oppositely so can be used to induce anesthesia.

Anticoagulants
The next class of drugs important for nclex exam is drugs ending with 'arin'. These are called
anticoagulants or blood thinners. Remember, arins are 'blood thinners'.
These prevent the formation of blood clots and thus can protect from stroke and heart attacks.
Examples are aspirin, heparin, and warfarin. These drugs are very popular and you should expect
to see at least one question from these drugs.
Important nursing considerations for anticoagulants:
 They are used in the treatment of pulmonary embolism, deep vein thrombosis, correction
of arial fibrillation which is also called cardioversion, and treatment of myocardial
infarction and some acute coronary diseases.
 So, when you are giving these anticoagulants the blood Platelet count, hemoglobin and
RBC count should be monitored well because this class of drugs decreases all these
parameters- please take note of this point as it is important for the exam.
 Apart from blood, you may also see conditions like bruising, blood in stools, and urine if
the dose is high.
 Also remember, patients with chronic renal disorders or renal diseases should take these
medicines with caution as these drugs are renally excreted.

After these common points for anticoagulants, let's now study individual drugs in little detail. We
will only study important points that might be helpful for your nclex exam. I will not be covering
the mechanism of actions and every interaction and side effect because that will be of no use to
memorize during examinations.
Remember the term 'Black box warning' when it comes to warfarin.
 A black box warning is the FDA's most stringent warning for drugs and medical devices
on the market.
 Warfarin can cause major or fatal bleeding. Perform regular monitoring of the international
normalized ratio (INR). Therapeutic INR range for the patient's taking Warfarin. 2-3 (this
value may be slightly higher if the patient is at high risk for clot formation 10 emergency.
 Changes in diet or the use of other drugs can alter their risk of bleeding and/or clotting,
sudden changes in dict such as spinach, cabbage, broccoli, collard greens, and lettuce which
contains vitamin k should be avoided.
 The antidote for warfarin is vitamin K and when you are on warfarin, you shouldn't take
any alcoholic beverages as it interferes with the drug.

Heparin
As with other anticoagulants, there is increased GI bleeding risk with heparin administration.
When it comes to the administration route, it can be administered only intravenously or
subcutaneously but not orally.Patients undergoing heparin therapy must be watched out for
Heparin-induced thrombocytopenia (HIT) which is an immune-mediated collapse in platelet
count. This can be very serious and fatal. The antidote to heparin is protamine.

Aspirin
Another anticoagulant is of class NSAIDs called Aspirin.
 There is a risk of peptic ulcer and bleeding with this drug so should be avoided in people
over 70 years of age.
Important points on aspirin poisoning:
 The aspirin poisoning can be treated with charcoal and alkaline diuresis.
 Salicylism or aspirin poisoning shows symptoms such as dizziness, ringing in the ears,
difficulty hearing, nausea, vomiting, diarrhea, mental confusion, and lassitude.
 Acute salicylate toxicity occurs at doses of 20-25 g in adults or 4 g in children.
 As aspirin is a salicylate, it has an increased risk of bleeding with other anticoagulants. So,
you shouldn't give aspirin together with other anticoagulants or NSAIDs as it may result in
an increased serum level of these drugs.

Antilepemics
Antilepemics reduce the level of lipids and lipoproteins in the blood. They reduce cholesterol. So
these are used in coronary artery diseases.
We know that epidemics can combine with cholesterol and can accumulate in the blood vessel so
this can cause blood flow obstruction causing heart attacks.
They end with-statin.
A common example is atorvastatin.
Other examples are:
 Fluvastatin.
 Lovastatin.
 Pitavastatin.
 Pravastatin.
 Rosuvastatin.
 Simvastatin.
Remember for nclex that there are various side effects with the use of these medicines.
 The common side effects are: rhabdomyolysis, sore muscles, hepatic impairment,
arthralgia, dizziness, upper respiratory infections, etc. Rhabdomyolysis or simply muscle
degeneration can be a life-threatening condition caused by muscle breakdown and muscle
death.
 Regarding muscle soreness giving niacin (vitamin B3) supplements along with a statin
medication may worsen muscle soreness.
 These drugs are given at night because most cholesterol is synthesized when dietary intake
is at its lowest.
 With grapefruit juice, the statins level in the blood can increase so you should not give
grapefruit and statins at the same time to a patient
 Statins can affect the liver. Statins should not be taken if you have severe liver disease or
if blood tests suggest that your liver may not be working properly.
 Note that these are contraindicated in pregnancy and childbirth. Give a special focus to this
point.
Medicines used for the cure of Integumentary System Diseases

Anti-acne
 The treatment approach for acne can be divided into the acne condition.
 Mild acne can be cared for with gentle cleansing and pharmacologically with topical
antimicrobials such as Dapsone, Benzoyl peroxide, clindamycin, etc, and retinoids.
 Moderate acne can be treated with oral antibiotics such as crythromycin, tetracycline,
Doxycyclines, etc.
 Severe acne can be treated with isotretinoin.

Psoriasis Treatment:
Psoriasis is a skin condition that causes flaky patches of skin which form scales. We will just focus
on important points to be remembered for your nclex examination.
Glucocorticoids
An important point to be remembered is this drug when used topically should never be used in
face and groin areas or any soft tissues. You can use glucocorticoids for milder psoriasis.

Calcipotriene
As this is a calcium derivative a higher dose for a long time can cause hypercalcemia.

Coal Tar
The method of treatment can cause skin irritation and cancer development

Keratolytics
The main keratolytics are salicylic acid and sulfur which are used in treatment of psoriasis.
Note one thing that salicylic acid is not applied in psoriasis covering a large surface area because
there is systemic effects with overdose. This can cause salicylism.
Another thing, the application of Sulphur can be done to treat acne and dandruff too other than
psoriasis.

Methotrexate
The main point you have to understand regarding methotrexate is, it can cause bone marrow
depression, diarrhea, and ulcerative colitis.
Anticancer/anti-neoplastic medicines
We will learn in brief about the drugs, side effects, interactions and nursing intervention, and
special care for clients on oncological medications in this lecture.
Anti-neoplastic agents are medicines that kill or stop the growth of neoplastic or cancer cells.
Having said that, during the treatment cycle, they can also affect or kill the normal cells. These are
usually combined with surgery, radiation therapy or other anti-cancer drug for the treatment of
cancer.
The dosage form is calculated comparing the BSA or total body surface area so that correct dosage
is given in response on the patient's body demand.
Let's see some important things about the nursing consideration and management for patients
taking antineoplastic drugs:
 As mentioned before, access the height and weight etc for BSA calculation so that you can
determine the dosage needed for patient.
 Monitor the blood cell count mainly the white blood cells and platelets because if the cell
count is low the patient may be very weak immunologically or may have bleeding
problems. As the patient who are under antineoplastic therapy may have bleeding
problems, you should avoid the intramuscular route of administration of the drug.
 Other thing to consider is vomiting. Patients under antineoplastic drug therapy have
vomiting as side effect. This can affect the drug absorption if the drug is expelled out in
vomiting. It is, for this reason, antiemetics of 5-HT3 antagonists granisetron and
ondansetron are given to the patient.
 As the immunity of the patient under therapy is very low, patient should be provided with
nutritious high calorie and proteinous diet.
 Pregnant and lactating mothers should never take the drug.
 Clients receiving IV chemotherapy have skin irritation and phlebitis-related problem so
any skin reaction should be properly monitored. Also, any leakage of the drug should be
properly addressed as this can cause skin necrosis. Heat or ice pack treatment should be
applied and the Health care worker should be immediately informed.
 The patients lose their hair during the treatment. You should tell the patient about this side
effect and to be ready for this psychologically.
 Any antiplatelet or agents that increase bleeding should be completely avoided. For
example, you should check the history if the patient is taking warfarin or aspirin
concurrently.
 There is a chance of biohazard and possible spills and leaks of the drug during treatment.
To avoid this first of all you should wear personal protective equipment or PPE and mask.
 As the patient undergoing treatment is immunocompromised, live vaccines should not be
administered. Also, the patient may get infertile irreversibly.
 Also, the contraceptive pills may not work well for the patient during treatment. You should
make them informed about all the side effects. A possible management for this could be
preserving the ova and the sperm.
 The patient is very immunologically compromised so our job is to ensure that the patient
is safe from contaminated environment or health. Tell the patient to avoid crowdy places,
wear mask and also to take good care of oral health to avoid growth of bacteria and fungus
and to avoid oral infections which may spread to other parts of the body.

Anti-metabolites.
The antimetabolites affect the cell division at the S phase by interfering with the protein synthesis
which is required for cell division.
Most of the drugs in this class ends with 'bine' or 'dine'. There are also many drugs in this class
that have different endings.
Let's see some examples of drugs from this class:
 Cytarabine
 Decitabine
 Gemcitabine
 Azacitabine
 Methotrexate
 Mercaptopurine
 Hydroxyurea
 Fluorouracil, etc

Methotrexate
 Methotrexate can be toxic if given at higher doses so as an antidote you can give
leucovorin. This is a very important point.
 Methotrexate is very common for photosensitive reactions, Hepatotoxicity,
Immunosuppression, GI toxicity, Skin reactions, and Bone Marrow Suppression.
 Like all other antineoplastic drugs, Methotrexate too can cause alopecia which means loss
of hair. You should tell the patients about the possible side effects. The hair loss is
reversible.
 It is phototoxic. It is for this reason that some care should be taken- such as walking in sun
with full sleeves or protective clothes, avoiding other drugs that have hepatic clearance
concomitantly, checking liver function tests for liver enzymes, etc.
 Similarly, it should not be given to people with hepatic diseases and old patients with renal
or hepatic diseases.
 The patient should also be instructed not to take alcohol during the treatment.
 Another antimetabolite drug, fluorouracil may also cause phototoxicity, alopecia, diarrhea,
and stomatitis (which means inflammation of the mucous lining of the oral cavity.
 Another drug of the antimetabolite family, mercaptopurine can cause hepatotoxicity. It can
also cause hyperuricemia.

Alkylating agents
Alkylating agents are compounds that work by adding an alkyl group to the guanine base of the
DNA molecule, preventing the strands of the double helix from linking as they should.
Examples:
Cisplatin
Carboplatin
Thiotepa
Busulfan
Altretamine etc.
 As with other antineoplastic drugs, they too can cause alopecia, pain, and phlebitis in the
IV site of injection and stomatitis.
 So as a nursing approach to caring for the patient, patients should be advised to take proper
care of their oral health and be ready for the side effects such as alopecia. Also, like other
anti-cancer drugs, there could be heavy vomiting so antiemetics can be taken.
 Some of the drugs of this class like Busulphan can cause stomatitis and a drug of this
family, Cisplatin can cause a decrease in the electrolytes of the body, ototoxicity, and other
hearing defects.
 Cisplatin is a platinum compound. Whenever you see this name in the nclex exam. It has
to do with some conditions that either harm the hearing or cause a decrease in electrolytes
causing hypokalemia or hypomagnesemia or hypocalcemia.

Antitumor Antibiotics
 These are the antibiotics that can interfere both with DNA and RNA replication of the tumor
cells.
 As with all other anti-cancer drugs, this too comes with side effects like nausea, vomiting,
stomatitis, hyperuricemia, Phlebitis, skin rashes, etc.
 Drugs like Doxorubicin and Daunorubicin are also cardiotoxic and may cause heart failure,
dysrhythmia, or cardiomyopathy.
 As a nursing management approach, you should watch out closely for ECG changes and
possible chances for dysrhythmia, heart failure, or peripheral edema.
Mitotic Inhibitors
 As the name suggests, mitotic inhibitors like vinblastine and vincristine inhibit mitosis and
prevent cell division.
 The side effects are hoarseness of the voice, decrease in white blood cell count, and
neurotoxicity so the patient feels numbness and tingling of fingers and toes which is the
side effect of vincristine,
 There is also the chance of paralytic ileus.
 Paralytic ileus occurs when the muscle contractions that move food through your intestines
are temporarily paralyzed. As a result, the patient may be constipated. Inform the patient
to take a fibrous diet and increase the amount of water intake.

Rheumatoid Arthritis Treatment


Basically, there is no treatment that fully cares about the diseases. However, 3 kinds of therapy are
seen to make the patient feel better. In extreme cases, there may be the need for surgical procedures and
replacement of the joints NSAIDs such as ibuprofen are common.
NSAIDs are To reduce the Gl upset, you should ask the patient to take the medicine only after taking food
and not on an empty stomach.

 Steroids such as Prednisone or dexamethasone. Note that prednisone can cause severe
hyperglycemia so you should monitor the glucose level regularly for diabetic and old
patients if placed in this therapy.
 Gold Salts- Gold salts decrease the progression of the disease. Side effects such as GI
disturbance and stomatitis are common. Drugs such as dimercaprol can enhance the
excretion of the Gold salts if severe side effects appear.
 DMARDs- slow down the destruction of joints by suppressing the body's immune system.
These are the first drug of choice if the condition is severe.
Examples are:
Methotrexate - one of the main side effects of methotrexate is bone marrow depression so this drug
shouldn't be used for a long time. It can also cause hepatic fibrosis.
Hydroxychloroquine- This can cause visual disturbances and also retinal damage if used for a long
time.
Leflunomide-It is a hepatotoxic drug and can cause various Gl symptoms and body rashes.
So, what could be the nursing interventions for rheumatoid arthritis?
 instruct the patient about the importance of physical mobility. This is very important as this
can maintain joint function. But too much mobility can cause pain. So, heavy exercise
should be avoided. You should instruct the patient to preserve the affected joints so that
further pain and effect can be reduced.
 instruct the patient about pain management. Despite NSAIDS used to manage the pain,
they can also take hot and cold therapy. Applying heat pads during morning stiffness can
help alleviate the stiffness. And cold can help reduce inflammation.
 instruct the patient about conserving energy. The patient can often feel tired and heavy and
also have to bear pain in the joints so heavy work and stress should be avoided. make the
patient feel better about their body image, as discussed before, the hands and feets joints
are destroyed and this can cause a crooked image of joints which can be socially disturbing.

EXAMPLE
Question: A nurse is accessing a client with deformed hands with complaints of pains and joint
stiffness in the morning. Upon investigation of the blood profile, it was confirmed that the patient
suffered from Rheumatoid Arthritis.
Which of the following advice would the nurse give the client?
 Avoid any exercises
 Avoid stress
 Take NSAIDS only after meals.
 make the patient feel better about their body image.

Answer: This is a multiple-option question, remember, you are asked for advice so there could be
multiple devices.
option 1 is not correct. The patient must be asked to conserve energy and take care of their joints
by avoiding heavy work but light exercise can help to ease the joint movement and reduce stiffness.
option 2 is correct. option 3 is also correct. taking NSAIDs such as ibuprofen on empty stomach
can upset the stomach.
option 4 is also correct. Rheumatoid arthritis progress as inflammation, formation of pannus, and
then deformation of the bones as the bones start fusing. This causes deformed hands and feet joints
so the patient may be feeling bad about his physical outlook.

Gout and Management


When the amount of uric acid content in the blood raises above normal for a long-term period, the
urate crystals start to get deposited in the joints resulting in pain and inflammation which is known
as gout. The disease progresses in definite phases such as Asymptomatic, Acute, Intermittent, and
Chronic. In the Acute stage, small joints such as that of the toe are inflamed, in intermittent
episodes of inflammation and pain are present between acute phases in the chronic, the urate
crystals are deposited in the organs.
So how would you Assess the disease?
One of the main identifying characteristics of this disease is the Tophi' which means a chalky
deposit of urate crystals causing formulation of nodules in the hands, feet, or other body parts.
There is a pain in the joints.
Note that deposition of urate crystals under the skin can cause rashes and pruritus and long-term
gout can also result in renal stone formation.

Nursing Management
Uric acid is the product of purine metabolism. So, you should advise the patient to take a less
proteinous and purine-containing diet. Avoid meat, wine, cheese, etc. Instead, you can take
vegetables like broccoli and this also helps to make urine green alkaline so that there is the
excretion of uric acid. These patients should avoid starvation and also avoid taking alcohol and
coffee because such activities can be the reason for acute gout attacks.
If a patient is having an acute attack with pain you should make him fie down with the affected
part raised.
To wash out uric acid from the body and to stop the formation of kidney stones, advise the patient
to take a lot of water at least 2 to 3 liters a day.
As medications for gout,
We can use NSAIDs for pain management and also some other medications like probenecid,
allopurinol, and colchicine.
Let's see some side effects and nursing considerations for managing clients with gout taking these
drugs.

Probenecid- Note that this drug should never be used with aspirin as salicylates can cause
elevation of the uric acid.

Allopurinol
One of the side effects of allopurinol is visual change or disturbance. You should advise patients
who are using allopurinol for a long time for a visual check-up. This drug should never be given
with hypoglycemic agents so care should be taken in a diabetic patient. This can increase the effect
of the drug so there can be a serious glucose fall. Also, this drug can increase the level of warfarin
so there could be extreme bleeding if used with warfarin concomitantly. Also, people taking
allopurinol shouldn't consume citrus fruits. Citrus fruits contain vitamin C which can aggravate
kidney stone formation with allopurinol.
So, making summary, All anti-gout drugs can have Gl effects and should be used with caution with
liver, cardiac, or GI-related diseases and older patients. They should never be taken with salicylates
such as aspirins.
Example
Question: A client has been prescribed Allopurinol and the nurse is giving instructions for the drug
usage to the patient. The patient is also taking hypoglycemic. Which of the following instructions
should she give to the patient?
1. Report any visual changes.
2. Take citrus fruits as natural supplements.
3. Regularly check the sugar level.
4. Do not take it with blood thinners.
Answer: The correct answer to this question is- Options 1,3 and 4.
This is a multiple options question so there could be more than 1 answer. You can guess that it is
a multiple options question as you are asked to select the correct instructions (plural).
Option 1 is right there is a chance of visual changes with allopurinol.
Option 3 is also right as allopurinol may increase the sugar level of hypoglycemia and the patient
is diabetic.
Option 4 is also correct because it can increase the blood level of blood thinners.
Option 2 is not correct. We should never use any sources containing too much vitamin c because
this can aggravate the formation of kidney stones.

Antithyroid Drugs
These are the drugs used to treat Hyperthyroidism. Symptoms of hyperthyroidism are:
 feeling tired all the time.
 sensitivity to heat.
 muscle weakness
 nervousness, anxiety, and irritability.
 hyperactivity .
 difficulty sleeping.
 diarrhea etc.
The main medicines falling in this category are:
 Iodide salts such as potassium iodide.
 Methimazole
 Propylthiouracil
 Carbimazole
 Radioactive iodine etc.

Important Points:
 As we discussed that some drugs of this class of drugs contain iodine (such as iodide salts
or radioactive iodine), Iodine can cause a metallic taste, swelling of gums, and pain in the
abdomen followed by diarrhea and vomiting.

 The blood cell count of the patients in this therapy should be regularly done because they
may be suffering from thrombocytopenia or agranulocytosis as a side effect.
 Note that for all thyroid disease conditions, you have to monitor the thyroid hormone level.
Sometimes, the dose of the drug may lead to hypothyroidism when taken for a long period.
So, monitoring the TSH and thyroid hormones is very important.
 You have to tell the patient not to take foods rich in iodine or salty foods as iodized salt
contain iodine, the level of salt intake should be reduced. Also, some drugs such as
acetylsalicylic acid may contain iodine hence, concomitant usage of these drugs must be
reduced.

Thyroid Storm and management


A thyroid storm is an acute and emergency case of hyperthyroidism. It is a life-threatening
situation. Undiagnosed and untreated hyperthyroidism for a long time causes thyroid storms. So,
as we learned in previous lectures, the management of hyperthyroidism is very important.
Cause: Thyroid surgery (when the thyroid hormone gets into the blood during surgery), may
aggravate thyroid storm. To reduce this, we need to give beta-blockers such as atenolol and
propranolol, iodides, antithyroid, etc. before thyroid surgery. This is also caused due to infections.
So, what are the symptoms of thyroid surge or thyroid storm?
 The patient may be in a coma or delirium (confusion).
 Tachycardia and hypertension.
 The patient may become feverish.
 The patient feels vomiting, restless, agitated, etc.

Nursing management procedures.


 First, see for airways.
 Check the vital signs of the patient.
 Give antithyroid drugs, beta-blockers, or glucocorticoids,
 To manage the fever, you can give antipyretics.
Example
A patient who left taking the antithyroid drug for a long time is admitted with symptoms of thyroid
storm. Which of the following symptoms can a nurse anticipate to observe for the patient? You
should select all the possible correct answers.
Your options are:
 The patient is feeling very cold and feverish.
 The patient has a heart rate of 115 bpm.
 The patient feels irritated, confused, and restless.
 The BP of the patient is 85/60.

The correct answer to this question is option number 1,2 and 3


. Option 4 is not correct. One of the symptoms of thyroid storm is systemic hypertension and
tachycardia. All the possible symptoms that occur in hyperthyroidism is also a symptom of thyroid
storm. It's just that it is an emergency medical situation.

Thyroid Drugs
Thyroid Drugs or thyroid hormones are used to treat hypothyroidism. They act as a synthetic
version of thyroid hormones and work the same as natural thyroid hormones would.
So, what are the thyroid hormones? Remember 2 important drugs in this class.
 Liothyronine sodium and
 Levothyroxine sodium.
The patient usually is intolerant to heat and may have decreased appetite with other symptoms
such as hair loss, sweating, and insomnia any symptom can lead to thyroid hormone deficiency so
a lot of conditions can lead to checking of thyroid hormone levels in the blood.
Let's see some nursing considerations for the patients taking these drugs as these could be very
important for nclex examination:
 The level of TSH and thyroid hormones must be closely monitored for people taking
thyroid drugs because the blood level can fluctuate over time. Taking thyroid drugs for a
long time without following a doctor's advice may cause hypothyroidism and cause thyroid
toxicity in which acute symptoms of hyperthyroidism such as palpitation, chest pain,
restlessness, etc are seen.
 The drugs must be taken on an empty stomach in the morning always. Instruct the patient
not to miss the dose.
 Mark the drug-drug interaction: Thyroid hormones are very interactive with many drugs
so their dose may need to be adjusted or concomitant use may be stopped.
 Antacids, Irons, multivitamins, etc decrease the absorption of these thyroid hormones so
must be stopped or not used concomitantly. There should be some time gap of 4 to 6 hours
between taking thyroid drugs and these medications.
 They can decrease the effect of antidepressants and also anticoagulants.
 They decrease the effect of cardiovascular drugs such as digitalis and also oral
hypoglycemic and insulin. Care must be taken if a patient is diabetic as well as suffering
from hypothyroidism.
 Anticonvulsants such as phenytoin also can decrease the effect of thyroid drugs. So you
should closely monitor the thyroid level if the patient is under thyroid hormone therapy
and also taking phenytoin and carbamazepine.

Burn Products
The major burn products are povidone-iodine (for minor burns, silver sulfadiazine, and mafenide
acetate.
Silver Sulfadiazine- The composition of this drug, silver is toxic to bacteria so this drug has a
broad spectrum activity against all gram-positive and gram-negative bacteria. Unlike the other
drug product like Mafenide used in burns, this doesn't cause metabolic acidosis.
One of the side effects of silver sulfadiazine is leucopenia so blood cell count and white blood cell
count can be done. Other side effects which are quite normal are bluish-grey discoloration of the
skin. Unlike other burn products, this is not irritating to the skin.
Mafenide Acetate- One of the widely used medicines in case of burn injuries is mafenide acetate.
The physicians try to keep the burn covered with mafenide acetate all the time. It is bacteriostatic
and reduces the growth of bacteria and is also water-soluble.
The main side effects are pain and rash and bone marrow depression or metabolic acidosis. As it
is water-soluble, it can diffuse to body parts and cause metabolic acidosis and hyperventilation as
a result.
If any such symptoms are seen, we have to wash the area and we may discontinue the use of
mafenide acetate for 1 to 2 days.

Example
A patient in an emergency with a second-degree burn is prescribed with topical application of
silver sulfadiazine. Which of the following regarding the use of silver sulfadiazine is not correct?
1. The drug has a broad spectrum activity against all gram- positive and gram-negative bacteria.
2. See for any symptoms of metabolic acidosis
3. Discoloration of the skin is common.

TEST YOUR KNOWLEDGE


Metronidazole
Which of the following would a nurse advise a patient who is on metronidazole therapy for the
treatment of giardiasis?
A. Do not use aftershave lotion.
B. Contact your healthcare provider immediately in you observe dark urine.
C. Take only a moderate amount of alcohol.
D. Contact your healthcare provider if you feel the metallic taste in your mouth.

This is a multiple-choice question for the correct use of metronidazole.


Remember, metronidazole is used to treat giardiasis as mentioned in the question along with other
GI-related disorders such as Amebiasis, colitis, etc.
As a nursing consideration and advice, remember, you would advise patients not to use alcohol or
any alcohol-containing products such as body sprays and aftershave lotions. The patient should
stop alcohol immediately 48 hours after treatment as it may cause disulfiram-like reactions.
A disulfiram-like drug is a drug that causes an adverse reaction to alcohol leading to nausea,
vomiting, flushing, dizziness, throbbing headache, chest and abdominal discomfort, and general
hangover-like symptoms.
The side effects of darkening of the urine or metallic taste in the mouth are normal. There is no
need to panic but the nurse can inform them about these general side effects to the patient too.

Cephalosporin
Which of the following advice from the nurse is not correct for the patient taking cefotetan?
A. -Present before / Tell the doctor if there is any history of use of penicillin.
B. Only take a moderate amount of alcohol when undergoing this treatment.
C. There could be Anorexia, vomiting, diarrhea, etc. You can take medicine with food.
D. You observe skin rash during the treatment. Severe allergic reactions should be reported.
This is quite an easy one to identify.
First of all, let's the properties of the drug that we are dealing with. Cefotetan is a Cephalosporin
class of drug. Please be cautious that like metronidazole, this drug too cannot sustain the reaction
with alcohol as it can cause a severe disulfiram-like reaction.
A disulfiram-like drug is a drug that causes an adverse reaction to alcohol leading to nausea,
vomiting, flushing, dizziness, throbbing headache, chest and abdominal discomfort, and general
hangover-like symptoms.
A patient under cephalosporin therapy may encounter similar side effects to that of penicillin such
as GI effects (vomiting, diarrhea, anorexia, etc.) but can also suffer from allergic or hypersensitive
reactions, Severe hypersensitive reactions must always be addressed properly. Allergic reactions
such as rashes and pruritis are quite common.
The GI discomfort can be relieved to some extent if the drug is taken with food. Also, concomitant
use of other cephalosporin drugs and interaction with penicillin can affect

Tetracycline
A nurse finds that one of the following activities of the patient is/are not the correct way for taking
the tetracycline drug.
1. She takes the medicine with a glass of lukewarm milk to eradicate the GI effects.
2. She takes sunbath to tone up her skin during the treatment.
3. She takes medicine only after 2 hours of use of other supplements.
4. She reports frequent incidences of diarrhea to the healthcare provider.
This is a multiple options question where there are 2 correct answers. As we are asked incorrect
options to choose from these 4 options. Option numbers 1 and 2 are the incorrect way of behaving
for someone undergoing tetracycline drug therapy.
Tetracycline can cause photosensitivity and can cause serious skin allergic reactions. A good way
to avoid this is avoiding walking under the sun or using sunscreen lotions or full-sleeve clothes.
As tetracycline can form complexes with Magnesium, Aluminium, or Calcium related products,
dairy products such as milk, supplements and antacids must be avoided.

Aminoglycoside
A patient undergoing treatment with gentamycin should be aware that-
A. There could be some hearing disturbances that could be irreversible.
B. The renal function tests must be done.
C. Any hearing problem such as vertigo, hearing loss, tinnitus, etc should be reported to the health
care provider.
D. The patient must report any past incidences of kidney-related discomfort or disease.
This is a multiple options question. This is the only question in the entire test scenario that we
covered so far where all the options are correct.
The most serious side effects of aminoglycosides like gentamycin are nephrotoxicity and
ototoxicity.
Nephrotoxicity means rapid deterioration in kidney function due to the toxic effect of medications
and chemicals.
Ototoxicity is when a person develops hearing or balance problems due to a medicine.
Note that ototoxicity is irreversible where as nephrotoxicity is at reversible side effect. To monitor
the occurrence and extent of nephrotoxicity, renal function tests that involve monitoring of BUN
and creatinine must be done.
Also, as this is a nephrotoxic drug, this may affect the health of the kidney and this is more serious
for kidney patients.

Fluoroquinolones
A patient who is taking ciprofloxacin asks for advice regarding the correct use of medication.
Which of the following advice would be incorrect?
A. Do not take Ciprofloxacin with milk and iron supplements.
B. Do not take Ciprofloxacin and walk out in the sun.
C. Discourage the use of Ciprofloxacin for old patients (> 60 years) and young children (<2 years).
D. Patients with Myasthenia Gravis should take a complete dose of ciprofloxacin.
The correct answer to this question is the last option. Ciprofloxacin is a fluoroquinolone. Drugs of
this category (Ciprofloxacin, Ofloxacin, Gatifloxacin, Moxifloxacin, etc. should not be taken with
iron supplements, milk products, or antacids as they can form complex compounds. As they are
photosensitive care while walking in the sun should be taken.
Also, old and young people must not be prescribed fluoroquinolone as it can affect the tendons
which are already weak in this population group.
The last option is incorrect. People with myasthenia gravis must not take any fluoroquinolones.
Fluoroquinolones are associated with negative implications for patients with myasthenia gravis
because they disrupt neuromuscular transmission.
Moxifloxacin
A patient is diagnosed with pneumonia infection. Which of the following actions are the most
likely actions that a nurse or a health care worker would try to access before administrating the
antibiotic 'moxifloxacin"?
1. See the blood profile for potassium level.
2. See the BUN level.
3. See the EKG QT interval before administration.
4. Access the thyroid functions.
The correct answers to these questions are option number 1 and 3.
Moxifloxacin is a drug under the fluoroquinolone family. Moxifloxacin is contraindicated for
patients with hypokalemia and patients with long QT intervals. Moxifloxacin can further prolongs
the QTc interval by 11.5-19.5 ms.
The rest of the options are incorrect.

Macrolides
A patient with a repeated history of Tonsilitis is prescribed erythromycin. The patient is already
taking warfarin. Which of the following is important to access in the blood profile?
1. See the warfarin level in the blood.
2. See the EKG QTs interval.
3. See the blood iron (hemoglobin) level in the blood.
4. See the T3 and T4 levels.
Erythromycin is a macrolide class of antibiotics. This drug can affect the warfarin levels in the
blood if a patient is also in warfarin therapy concurrently. So, the Ist option is the correct option
here.
The second option is also correct. Erythromycin can also increase the QT level and this can cause
cardiac dysrhythmia. The other two options have no direct relation with erythromycin. These are
just the options to confuse you.

Warfarin
A patient who is a stroke survivor has been prescribed warfarin sodium. Which of the following
actions from a nurse is not the correct intervention?
1. Note if the patient is taking any drug or any antibiotic concurrently
2. Monitor PT and INR.
3. Teach the patient importance of increasing/adopting green leafy vegetables in their diet.
4. Tell patients to take good care of their oral health.
The correct option (incorrect answer) for this question is an option no 3. Green Leafy vegetables
contain potassium. Potassium acts as an antidote to warfarin. So, increasing green leafy vegetables
can affect the warfarin levels in the blood.
Option 1 is correct so this is not the answer to this question. (We are looking for incorrect answers).
PT stands for Prothrombin time and INR stands for International Normalized Ratio. The
concentration of these parameters is important to measure as PT/INR test helps diagnose the cause
of bleeding or clotting disorders. It also checks to see if a medicine that prevents blood clots is
working the way it should.
The patient with warfarin has a bleeding tendency from gums. Information to take good care of
oral health and brushing with soft-bristle brushes can be beneficial to patients.

Calcium Channel Blocker (Verapamil)


A patient with high blood pressure is prescribed verapamil (80 mg t.i.d.). Which of the following
activity could pose a serious risk to the patient?
A. The patient is taking green leafy vegetables more.
B. The patient has not yet quit smoking.
C. The patient consumes wine and red meat regularly.
D. The patient is taking a banana shake with grapefruit juice.
The last option is the correct option in this scenario. Verapamil is a calcium channel blocker.
Grapefruit juice interacts with some calcium channel blockers and increases the level of the
medicine in the blood other options are just taken for your confusion.

Nitroglycerin
A patient with acute angina is administered nitroglycerine sublingual tablets. Which of the
following is not a correct nursing intervention to care for such patients?
1. Ask the patient to carry the medicines with them every time.
2. Ask the patient to abstain from alcoholic beverages.
3. For angina pain do not give more than 3 tablets every 5- minute interval.
4. Ask patients to drink plenty of water after swallowing the tablets.
The incorrect procedure is option 4 hence it is the correct answer to this question. Nitroglycerin
comes as a sublingual tablet to take under the tongue. The tablets are usually taken as needed, so
there could be a need at any time so the patient must be instructed to always carry the tablets with
him her. It is generally given 5 to 10 minutes before activities that may cause attacks of angina or
at the first sign of an attack. Care should be taken so that no more than 3 tablets are given. If the
angina pain doesn't subside, the health care worker must be called. The tablets can be given at 5
minutes intervals if the patient still complains the pain.
Also, the patient must not take any alcoholic beverages. If he/she takes alcohol while under
nitroglycerin therapy, it can cause a rapid heartbeat (tachycardia), sudden changes in blood
pressure, dizziness, and fainting.

Benzodiazepine
A patient who is currently under benzodiazepine for the treatment of insomnia has to know that:
1. The patient should take the medication in the morning on an empty stomach.
2. It is not safe for the patient to drive while the medication is on.
3. The medicament may be coated for long drug release, it should not be broken or chewed before
administration.
4. He/She should be taking only a moderate amount of alcohol.
This is a multiple-option question. The correct answer to this question is 2 and 3. The patient is
already feeling insomnia for which he may be feeling lethargy. Further, the medicine makes him
fall sleepy, and drowsy because of which it is unsafe to do activities that require attention. It is
also, for this reason, the medicine is administered before sleeping (at night) so option I is incorrect.
Option 3 is also correct as to prolong the release of the drug sometimes the drugs may have a
special coating around them also called an enteric coating. When the pill is broken or chewed, this
coating may be broken so the effect of the drug working for a long time is lost. Option 4 is also
incorrect- The patient should abstain from taking alcohol as it further aggravates sedation.

Insulin
Which of the following knowledge about the use of insulin for administration to the patient is not
correct?
1. We can combine humulin with short-acting insulin (lispro).
2. The administration area must be massaged well for good distribution and ease of injection pain.
3. The insulin-prefilled syringes must always be kept vertically down (with the needle pointing
towards the ground) while storing.
4. Short-acting insulins are given through the Subcutaneous route.
The correct answer to this question is option no.1.
NPH or Humulin can be mixed with short-acting insulins. While mixing care should be taken that
short-acting insulins (lispro) should be drawn out before NPH as NPH is cloudy and may
contaminate the mixture. Also, note that of all the insulin only NPH looks like the cloudy
suspension insulin and should be shaken and mixed well within palms before administration. The
administration area is usually the abdomen but can be rotated but should never be massaged. The
insulin syringe is generally kept in the refrigerator but should be kept vertically up. Also, note that
short-acting insulin is only the insulin that can be given intravenously (not subcutaneously) so this
option is also wrong.

Metformin
Which of the following instruction given to the patient about the use of metformin is not correct?
A. Avoid alcohol.
B. Take folic acid as a supplement.
C. Take vitamin B12 supplements or food rich in vitamin B12.
D. If a dose is missed, add the previous dose and maintain the previous dose as
prescribed/scheduled.
The patient under metformin should avoid alcohol. Combining alcohol and metformin has risks.
Drinking alcohol can make it more likely to develop lactic acidosis, a health complication that
needs medical attention.
Vitamin B12 is an especially important vitamin for maintaining healthy nerve cells, and it helps in
the production of DNA and RNA, the body’s genetic material. Vitamin B12 works closely with
vitamin B9, also called folate or folic acid, to help make red blood cells and to help iron work
better in the body.
The dose should never be skipped or combined so the last option is not correct.

Sulfonylurea
Which of the following habit of a patient under sulfonylurea therapy should be changed?
1. The patient carries candies with him.
2. The patient fears that ingestion of alcohol may harm him.
3. The patient checks the sugar level multiple times a day.
4. The patient skips meals when he feels GI upset.
The correct answer is option 4 as skipping a meal is strictly prohibited for such patients. Skipping
a meal may cause rapid hypoglycemia with may be fatal in certain cases. It is a good habit to check
the sugar level multiple times a day or to abstain from taking alcohol too. When the sugar level is
low, candies can provide a quick way of increasing the sugar. So patients can carry candies to
prevent the hypoglycemic effects.

Anti-gout
A patient explains his dietary habits for the treatment of gout. Which of the following regarding
dietary habits during the use of probenecid is not necessary?
1. The patient is taking the medicine with bread and milk as it helps to case his GI discomfort.
2. The patient is avoiding alcoholic products.
3. The patient is taking foods rich in high protein.
4. The patient is taking enough fluids and water.
The correct answer for this is option number 3. The patient should avoid a high protein diet as far
as possible. Since high-protein foods tend to contain large quantities of purines, patients with gout
are generally advised to avoid food sources of protein, including meat, seafood, soy, and legumes.
Drinking too much alcohol may increase the amount of uric acid in the blood and lessen the effects
of this medicine.
Option 1 is correct. The interaction with milk is not reported and taking medicine with food helps
to ease the GI discomfort. To help manage gout, it's also important to stay hydrated by drinking
plenty of water so option number 4 is also incorrect.

Tricyclic antidepressants
A patient with suicidal attempt history and under tri-cyclic antidepressants is seeking advice for
the correct use of medication. Which of the following best advice for the caretaker of such patients?
1. Administer the medicine at bedtime.
2. Stop the medicine immediately if the patient complains of orthostatic hypotension.
3. Gradually, give the patient free space as the patient's depressive condition improves.
4. Advise the patients on the importance of people and family.
The correct answer to this question is option no. 1. Tricyclic antidepressants are given at bedtime
because it helps to avoid the side effects of sedation to patients. The patients under this therapy
should not be driving or getting involved in things that need attention.
Option 2 is incorrect. Orthostatic hypotension is a common phenomenon or side effect of the drug.
To avoid this avoid rising immediately and teach the patients to cope with orthostatic hypotension.
Option 3 is also wrong. Contrary to what the option answer suggests, we are given in the question
that the patient has a history of suicidal attempts too. As the patient begins to feel better, the suicidal
tendency increases instead of decreasing. Any abrupt mood change should be closely watched out
for.
Option 4 is also wrong. We are searching for the best advice. Only advising people about the
importance of family or making them feel good may not be the best option if other options exist.

Antihistamine
A patient with an acute allergic reaction is prescribed diphenhydramine. Which of the following
activities must the patients know about the safety of the drug?
1 The patient should not drive.
2. The patient should not take water frequently.
3. The patient should not have undergone any skin tests within a week of treatment.
4. The patient should not take alcohol.
The correct answer to this question is option 1 and option 4.
Diphenhydramine is an antihistamine. More accurately, it is a First Generation Antihistamine. The
side effects of first- generation antihistamines are anticholinergic effects such as dry mouth,
sedation, constipation, difficulty urinating, and loss of accommodation. To stop dry mouth, the
patient may need to take water frequently. Also, as one of the side effects is sedation the patient
should not get involved in activities that demand high attention such as driving. The patients
shouldn't take alcohol too. As the drug already has an anticholinergic effect and can make you
sleepy and sedate, alcohol can worsen this condition and take the patient into deep sedation.
Antihistamines will affect the results of skin prick tests and need to be stopped before the testing
is completed to get accurate results. If a patient is taking an oral antihistamine the medicine should
be stopped a week (or at least 4 days) before his/her appointment.

Acetaminophen
Which of the following is true about the use of acetaminophen?
1. It can cause Reye's syndrome in a large dose.
2. It should not be taken with milk or with food.
3. It has a little anti-inflammatory effect.
4. It has a little anti-rheumatic effect.
The correct answer to this question is option one. Reye's (Reye) syndrome is a rare but serious
condition that causes swelling in the liver and brain. Aspirin is generally taken to cause Reye's
syndrome.
It can be taken with milk/food to reduce GI irritation. We have to understand that acetaminophen
has antipyretic and analgesic properties only. It cannot be used to treat inflammation or can be
used for anti-rheumatic actions.

Oxytocin
A pregnant woman is administered oxytocin to induce term labor. The nurse immediately stops the
infusion. All of the given conditions may be the true reasons for stopping oxytocin infusion except
1. The uterine pressure is greater than 20 mm Hg
2. There is an alteration in fetal heart rhythm.
3. The mother is complaining of intense pain.
4. The contractions are occurring frequently (every 2 minutes).
]The correct answer to this question is option no, 3 as it is incorrect. All the other options are
correct. Generally, the oxytocin infusion is stopped and a healthcare worker is called in the
following scenario:
 Frequent contractions.
 Alteration in fetal heart rate.
 Increase in uterine pressure (more than 20mm Hg)
 Contractions are lasting longer (more than a minute)

Fentanyl
A patient who is opioid tolerant is administered a Transdermal patch of Fentanyl to help alleviate
pain. Which of the following instruction should be given to such patients is not correct?
1. Avoid too much exercise.
2. Keep the patch area warm with heating sources.
3. This can cause dependency on long use.
4. The patient may suffer from respiratory depression.
Option 2 is not correct and hence the answer to the question in this situation.
The medicine from the patch may release fast on the administration of heat or any activity that
raises the body temperature such as hard exercises so heat and hard physical exercises must be
avoided.
Option 3 is correct as there is dose-related dependence on long use.
Option 4 is also correct. Fentanyl causes respiratory depression although slow-release devices such
as patches are not very harmful. Note that respiratory depression can be reversed with naloxone.

Digitalis
The ECG of the patient shows signs of atrial fibrillation. The health care provider suggested
treatment with digitalis to the patient. Which of the following is a nursing procedure to follow in
this case?
1. Monitor the pulse of the patient and serum level of digitalis
2. Administer IV boluses to the patient.
]3. Watch out for any signs of toxicity.
4. In case of a missed dose, give the double dose and maintain the normal dose as suggested
afterward.
Only option number 1 and 3 are correct in this scenario. Digitalis is used to improve cardiac output
and treat atrial fibrillation and atrial flutter. The IV infusion should be given slowly for over 5
minutes.
One of the main adverse effects of digitalis is digitalis toxicity due to overdose.
Warning signs of overdose are confusion, loss of appetite, nausea, vomiting, diarrhea, or vision
problems. Other signs of toxicity or overdose are changes in the rate or rhythm of the heartbeat
(becoming irregular or slow), palpitations (feeling of pounding in the chest), or fainting.
The antidote used in such a case is digoxin immune Fab (Digibind). Digibind binds and inactivates
digoxin. The dose must never be skipped or never doubled if there is a miss.

Antihyperlipidemic
A woman with an altered triglyceride level is prescribed atorvastatin. Which of the following
advice is the best advice for such a patient?
1. Please refer to a healthcare worker if there is muscle pain and weakness.
2. Please do not have plans to conceive a child (if she is of child-bearing age).
3. Take medicine in the morning on empty stomach.
4. Do not let to lose weight.
These classes of drugs help to lower the cholesterol level. It helps to decrease the LDL and increase
the HDL level which is also called good cholesterol. The medicine is taken in the evening. A
healthy diet and Exercise also help to maintain the level of triglycerides and reduce the LDL level
so it must also be accompanied by the treatment.
One of the side effects of antihyperlipidemic (Atorvastatin, Pravastatin, Simvastatin, etc.) is
myopathy which is muscle weakness due to dysfunction of muscle fiber. It could be accompanied
by symptoms such as muscle cramps, stiffness, and spasm.
The other side effect of serious cases is rhabdomyolysis. This is a serious medical condition that
can be fatal or result in permanent disability. It occurs when damaged muscle tissue releases its
proteins and electrolytes into the blood. These substances can damage the heart and kidneys and
cause permanent disability or even death.
The medicine affects the liver so any patient with liver dysfunction should take the drug with
caution. It is for this reason, the liver function tests of such patient are monitored.
Also, please note that this drug should be avoided by child- bearing mothers or pregnant women
because it can harm the growing fetus.

Benzodiazepine
The healthcare worker is preparing the patient for conscious sedation of the patient and gives a
dose of benzodiazepine for the anesthetic procedure. Which of the following is not a correct
nursing procedure to follow?
1. Watch out for any cardiac or respiratory changes throughout the sedation period.
2. Tell the patient about the surgical procedure in brief and pacify him to ease the fear after the
operation.
3. Monitor vital signs and BP.
4. Watch out for the progress of sedation. If required give additional doses slowly after 2-3
minutes.
The incorrect answer and hence the correct option for this question is option no. 2.
The sedation starts quite early within 1-2 minutes and even after the operation the patient won't be
able to recall anything said to him so there is no question of pacifying the patient and telling him
how the procedure went.
There could be abrupt respiratory changes, cardiac effects, etc. so monitoring the both cardiac and
respiratory state of the patient is vital.
The dose given to the patient may cause a cardiac or respiratory problem so the drug should be
given slowly with monitoring and additional dosage must be given only after conscious
observation of sedation for some minutes.

Atropine
The ECG of a patient shows Atrioventricular Block (AV) Block. The health care worker prescribed
Atropine. Which of the following actions is not a correct way of dealing with such a condition?
1. Keep the patient hydrated by letting him sip fluids and water.
2. Keep account of the number of urinary calls.
3. Watch out for the history of BPH (Benign Prostatic Hyperplasia)
4. Administer the next dose if tachycardia is not relieved.
Atropine is used to increase the heart rate and hence can be used in bradycardia and AV blocks.
Atropine further aggravates tachycardia. Hence option 4 is not the correct way of dealing with
such a problem. However, there are some serious side effects and contraindications that we have
to consider.
Side effects of atropine include dry mouth, blurred vision, photophobia, tachycardia, flushed skin,
constipation, difficulty with urination, inability to perspire appropriately, delirium, or coma.
Option 1 and 2 are measures to keep off these side effects.
One of the side effects is bladder obstruction. Benign prostatic hyperplasia (BPH) is another name
for prostate gland enlargement. It is a common condition as men get older. An enlarged prostate
gland can cause uncomfortable urinary symptoms, such as blocking the flow of urine out of the
bladder. The side effect of Atropine (difficulty in urination) makes this condition even worse.

Aspirin (Salicylate Poisoning)


A child who ingested some pills of aspirin is admitted with signs of vomiting, nausea, and difficulty
in breathing Which of the following actions are required for emergency nursing procedures?
1. Gastric Lavage should be done immediately.
2. Ventilate and provide oxygen to the patient.
3. Prepare for dialysis if necessary.
4. Keep the patient covered with warm clothes.
The correct options for this question are options 1,2 and 3.
Salicylate poisoning is a common case and medical emergency of poisoning and generally happens
in children when they ingest large doses or adults who take a large dose to relieve any pain.
Remember just 4g of salicylate is lethal to a child. It is for this reason, the medication should be
kept safe and children should be kept away from its use.
The common signs of salicylate poisoning are: Early signs- Nausea, vomiting, diaphoresis,
tinnitus, vertigo, hyperventilation, tachycardia, and hyperactivity. Late signs- Delirium,
hallucinations, convulsions, lethargy, and stupor.
As a first aid, you should perform gastric lavage and use activated charcoal, treat dehydration with
electrolyte therapy, infuse bicarbonates, and try to cool the temperature. Sometimes for high-dose
ingestion, dialysis may also be necessary.

Isoniazid
A patient was diagnosed with pulmonary tuberculosis. Which of the following activity of a patient
under isoniazid treatment for latent tuberculosis needs rectification?
1. The patient takes the pill after the meal.
2. The patient takes vitamin B6 as a supplement.
3. The patient abstains from taking alcohol.
4. The patient is also taking other antitubercular drugs along with isoniazid.
The habit that needs correction is taking the drug after a meal. Isoniazid must be taken on an empty
stomach. The second option is correct. Vitamin B6 helps to prevent the development of peripheral
neuropathy.
The third option is also correct. The drug itself is hepatotoxic hence consumption of alcohol can
aggravate the situation. The fourth option is also correct. The drug is used in combination with
other antitubercular drugs. For mnemonics, please memorize STRIPE
ST- Streptomycin
R-Rifampin
I- Isoniazid
P-Pyrazinamide
E- Ethambutol
Morphine
A patient is prescribed morphine sulfate for the alleviation of the pain of surgery. Which of the
following nursing consideration sounds justifiable for the care of such a patient?
1. Do not give the medicament if the respiration rate is very low.
2. It is a safer choice of pain management for people with renal dysfunction.
3. It is a safer choice for pain management who encounter orthostatic hypertension.
4. Atropine reverses the effect of morphine in case of toxic doses.
The correct answer to this question is option no. 1.
Morphine depresses the respiratory system hence if the respiratory rate is lesser than 12 per
minute, it is safe not to give morphine. It is, for this reason, morphine is not usually administered
to old people (>60 years) and younger children, and patients who have underlying respiratory
disorders such as COPD and Asthma.
It is contraindicated for people with renal and hepatic disorders and may cause orthostatic
hypertension. The antidote for Morphine is Naloxone not Digoxin.

You might also like